Nervous System + Tissue (Ch. 11)

Pataasin ang iyong marka sa homework at exams ngayon gamit ang Quizwiz!

True or false: Sensory neurons conduct impulses from the spinal cord to a muscle or a gland

False

Why does regeneration of the action potential occur in one direction, rather than in two directions?

The inactivation gates of voltage-gated Na+ channels close in the node, or segment, that has just fired an action potential.

electrical and chemical signals

cells communicate within the nervous system via

ependymal cells

cells that line cerebrospinal fluid-filled cavities of the brain and spinal column; form permeable barrier between cerebrospinal fluid and tissue fluid bathing CNS cells; range in share from squamous to columnar; may be ciliated

Which reflex has a contralateral component?

crossed-extensor

Cardiovascular effects of the sympathetic division include all of the following EXCEPT ________.

dilation of the blood vessels serving the skin and digestive viscera

Which of the following is NOT a result of parasympathetic stimulation?

dilation of the pupils

lines brain cavities

ependymal cell

Nervous System

master controlling and communicating system of the body

The two longitudinal ridges on the medulla oblongata where many descending fibers cross over are called the ________.

pyramids

The minimum voltage that is required to generate an action potential is called the

threshold voltage

The posterior side of the thigh, leg, and foot is served by the ________ nerve.

tibial

During the depolarization phase of an action potential, the permeability of the membrane

to Na+ is greatly increased

The small space between the sending neuron and the receiving neuron is the

synaptic cleft

A myelinated nerve fiber will appear ______, while an unmyelinated fiber appears ______.

white; gray

What is the magnitude (amplitude) of an action potential?

100mV

A) Brachial plexus B) Lumbar plexus C) Cervical plexus D) Sacral plexus 22) The obturator and femoral nerves branch from this plexus. 23) Striking the "funny bone" (ulnar nerve) may cause injury to a nerve of this plexus. 24) Trauma to a nerve of this plexus may cause wrist drop. 25) A fall or improper administration of an injection to the buttocks may injure a nerve of this plexus. 26) The phrenic nerve branches from this plexus.

22) B 23) A 24) A 25) D 26) C

2) Nerve impulses leading to the brain carry information about cool temperatures on the skin. The nerve fibers sending these signals will most likely belong to which division of the nervous system? A) sensory (afferent) division B) somatic nervous system C) sympathetic division D) parasympathetic division

A) sensory (afferent) division

17) Fibers that enter and leave the sympathetic trunks without synapsing form structures called ________. A) gray rami communicantes B) splanchnic nerves C) white rami communicantes D) spinal nerves

B) splanchnic nerves

Bundles of neurons in the central nervous system are called tracts.

Bundles of neurons are called tracts in the central nervous system. In the peripheral nervous systems, bundles of axons are called nerves.

A. True

Bundles of neurons in the central nervous system are called tracts. ANSWER: A. True B. False

Branch of an axon

Collateral

What does the axon (nerve fiber) do?

Conduct impulses away from the cell body

What do dendrites do?

Conduct impulses toward the cell body

Sensory (afferent) Neuron

Conducts impulses from receptors in peripheral body parts into the brain or spinal cord

Neuron process with many branches that conducts an action potential (impulse toward the cell body)

Dendrite

The neuropeptides that act as natural opiates are

Enkephalins

Specialized mass of neuron cell bodies outside the brain or spinal cord

Ganglion

A) Calcium ions

Match: Calcium Ions

Multipolar, Bipolar, or Unipolar? Sensory neuron

Most unipolar; some bipolar

Which of the following is a subdivision of the autonomic nervous system?

Sympathetic division

Where does nervous tissue occur?

The brain, spinal cord, and peripheral nerves

In this simulation, ___________________ will be used to stimulate the axon.

Voltage

Ependymal cells line many open cavities in the central nervous system (CNS). Ependymal cells have cilia on the side of the cell that face these openings. What is the most likely function of these ciliated cells?

Yes, cilia are an indication that the cells they appear on are moving fluids past the surface of the cells.

When the stimulus voltage is increased,

a greater-than-threshold depolarization results and sodium permeability into the cell increases to overcome the potassium exiting

Which of the following structures is not part of the central nervous system?

a nerve

If a post synaptic cell is stimulated to threshold by temporal summation this implies that ________.

a presynaptic neuron is initiating frequent EPSP

Which of the following cranial nerves carries only motor information?

abducens

Microglia Cells

are small and ovoid with relatively long "thorny" processes. Their processes touch nearby neurons, monitoring their health, and when they sense that certain neurons are injured or are in other trouble, the microglial cells migrate toward them. Where invading microorganisms or dead neurons are present, the microglial cells transform into a special type of macrophage that phagocytizes the microorganisms or neuronal debris.

The central nervous system consists of

brain and spinal cord

The term central nervous system refers to the

brain and spinal cord

tracts

bundles of neuron processes in CNS

What division of the nervous system is most specifically responsible for voluntary motor control? a) sympathetic nervous system b) parasympathetic nervous system c) somatic nervous system d) central nervous system

c

Which of the following correctly describes a graded potential? a) it travels long distances b) it has a depolarization, repolarization and hyperpolarization phase c) it can have amplitudes of various sizes d) it is initiated by voltage changes in the membrane

c

Receptors that bind acetylcholine at the postsynaptic membrane are

chemically regulated channels

What type of current flows through the axolemma during the steep phase of repolarization

chiefly a potassium current

What type of current flows through the axolemma during the steep phase of repolarization?

chiefly a potassium current

endocrine system

communicates by means of chemical messengers (hormones) secreted into the blood

Which of the following is the mildest consequence of traumatic brain injury

concussion

The ________ includes the thalamus, hypothalamus, and epithalamus.

diencephalon

neurons (nerve cells)

excitable cells that transmit electrical signals

parasympathetic division

functional division of autonomic nervous system that conserves energy and promotes house-keeping functions during rest

The velocity of nerve impulse conduction is greatest in

heavily myelinated, large-diameter fibers

The velocity of nerve impulse conduction is greatest in:

heavily myelinated, large-diameter fibers

Which of the following is not a function of the autonomic nervous system?

innervation of skeletal muscle

Which of the following is used to block pain

lidocaine

Which of the following is NOT a functional region of a neuron

medullary region

the blood-brain barrier is effective against

metabolic waste such as urea

CNS phagocytes

microglia

CNS phagocyte

microglial cell

astrocytes

most abundant, highly branched glial cells found in CNS; cling to neurons, synaptic endings, and capillaries; support and brace neurons; control chemical environment around neurons

A fracture of the ethmoid bone could result in damage to which cranial nerve?

olfactory

A shallow groove on the surface of the cortex is called a ________.

sulcus

The all-or-none phenomenon as applied to nerve conduction states that the whole nerve cell must be stimulated for conduction to take place.

F

The concentrations of which two ions are highest outside the cell.

Na+ and Cl-

Which of the following correctly states the direction followed by the specified ions when their voltage-gated channels open?

Na+ ions move into the axon; K+ ions move out.

B. False

Na+ leaks through neuron membranes faster than K+ because of the properties of the non-gated leak channels. ANSWER: A. True B. False

D) parasympathetic division

Nerve impulses are sent to slow the heart's rate of contraction. The nerve fibers sending these signals will most likely belong to which division of the nervous system? A) sensory (afferent) division B) somatic nervous system C) sympathetic division D) parasympathetic division

A) sensory (afferent) division

Nerve impulses leading to the brain carry information about cool temperatures on the skin. The nerve fibers sending these signals will most likely belong to which division of the nervous system? A) sensory (afferent) division B) somatic nervous system C) sympathetic division D) parasympathetic division

Name the glial cell at F.

Schwann cells carry out myelination of the peripheral nervous system (PNS)

What is the function of ependymal cell?

Secrete and assist in circulation of cerebrospinal fluid

Transmits impulse into brain or spinal cord from receptors

Sensory (afferent) neuron

In myelinated axons the voltage-regulated sodium channels are concentrated at the nodes of Ranvier.

T

Cell bodies of sensory neurons may be located in ganglia lying outside the central nervous system.

True

During depolarization, the inside of the neuron's membrane becomes less negative

True

Large-diameter nerve fibers conduct impulses much faster than small-diameter fibers

True

Myelination of the nerve fibers in the central nervous system is the job of the oligodendrocyte

True

Neurons in the CNS are organized into functional groups

True

Opening K+ or Cl- channels in a postsynaptic membrane would produce an inhibitory postsynaptic potential (IPSP). True or False

True

Some neurotransmitters can be either excitatory or inhibitory depending upon the receptor.

True

The overlapping functions of the nervous system are sensory input, integration, and motor output.

True

True or False: Astrocytes have contacts between blood vessels and neurons in the CNS

True

motor output

activation of effector organs (muscles and glands) produces a response

Where in the neuron is an action potential initially generated?

axon hillock

the anatomical region of a multipolar neuron has the lowest threshold fro generating an AP is the

axon hillock

Sodium and potassium ions can diffuse across the plasma membranes of all cells because of the presence of what type of channel?

leak channel

The channels that provide for the movement of potassium in the resting neuron are

leakage

Which of the following allow the movement of potassium through the neuronal membrane? Which of the following allow the movement of potassium through the neuronal membrane?

leakage channels and voltage-gated potassium channels

sympathetic, parasympathetic

name the two functional divisions of the autonomic nervous system that work in opposition to each other

sensory division, motor division

name the two functional divisions of the peripheral nervous system

Regulation by the nervous system provides

swift but brief responses to stimuli

Preparing the body for the "fight-or-flight" response is the role of the ________

sympathetic division

Norepinephrine is a neurotransmitter at some of the synapses in the

sympathetic system

neurons

structural units of the nervous system; large, highly specialized cells that conduct impulses

Degeneration of the dopamine-releasing neurons of the ________ is the ultimate cause of Parkinson's disease.

substantia nigra

Which of the following is NOT a diencephalon component?

superior colliculus

schwann cells

surround all peripheral nerve fibers and form myelin sheaths in thicker nerve fibers; similar function as oligodendrocytes

satellite cells

surround neuron cell bodies in the PNS; function similar to astrocytes of CNS

Axon diameter and degree of myelination determine nerve impulse conduction velocity.

true

Enkephalins and endorphins are peptides that act like morphine

true

If bacteria invaded the CNS tissue, microglia would migrate to the area to engulf and destroy them.

true

In myelinated axons the voltage-regulated sodium channels are concentrated at the nodes of Ranvier.

true

Reflexes are rapid, automatic responses to stimuli.

true

Synaptic transmission is only one way.

true

The action potential is caused by permeability changes in the plasma membrane

true

The oligodendrocytes can myelinate several axons

true

Unipolar neurons have axons structurally divided into peripheral and central processes

true

30) After axonal injury, regeneration in peripheral nerves is guided by ________. A) Schwann cells B) Wallerian cells C) Golgi organs D) dendrites

A) Schwann cells

65) Visual processing in the thalamus does not contribute significantly to ________. A) high-acuity vision B) night vision C) movement perception D) depth perception

B) night vision

31) Which of the following is not a chemical class of neurotransmitters? A) biogenic amine B) nucleic acid C) amino acid D) ATP and other purines

B) nucleic acid

39) Mixed cranial nerves containing both motor and sensory fibers include all except which of the following? A) facial B) olfactory C) oculomotor D) trigeminal

B) olfactory

55) An essential part of the maculae involved in static equilibrium is (are) the ________. A) spiral organ (of Corti) B) otoliths C) cupula D) scala media

B) otoliths

18) Which of the following nerves does not arise from the brachial plexus? A) radial B) phrenic C) ulnar D) median

B) phrenic

28) Which receptors adapt most slowly? A) nociceptors B) pressure receptors C) smell receptors D) touch receptors

B) pressure receptors

29) Which category of memory is involved when playing the piano? A) motor B) procedural C) emotional D) declarative

B) procedural

The all-or-none phenomenon as applied to nerve conduction states that the whole nerve cell must be stimulated for conduction to take place.

False

The nodes of Ranvier are found only on myelinated, peripheral neuron processes

False

A) C B) E C) A D) B E) D 1) White fiber tracts. 2) Thalamus. 3) Pons. 4) Hypothalamus. 5) Medulla Oblongata.

1) C 2) D 3) E 4) A 5) B

43) Which statement about epilepsy is most accurate? A) Epilepsy is often genetically induced but also frequently caused by head trauma, stroke, infection, and tumor. B) Absence seizures typically begin in adolescence and are often severely disabling. C) The aura in tonic-clonic seizures typically occurs as the patient regains consciousness. D) During seizures, sensory messages are processed normally but responses are blocked.

A) Epilepsy is often genetically induced but also frequently caused by head trauma, stroke, infection, and tumor.

29) Nerves that carry impulses toward the CNS only are ________. A) afferent nerves B) mixed nerves C) efferent nerves D) motor nerves

A) afferent nerves

28) The process of linking new facts with old facts already stored in the memory bank is called ________. A) association B) automatic memory C) rehearsal D) long-term memory

A) association

6) What type of cells line the ventricles of the brain? A) ependymal cells B) neurons C) astrocytes D) epithelial cells

A) ependymal cells

17) Which of these would you NOT find in the cerebral cortex? A) fiber tracts B) interneurons C) dendrites D) cell bodies

A) fiber tracts

15) The central sulcus separates which lobes? A) frontal from parietal B) frontal from temporal C) parietal from occipital D) temporal from parietal

A) frontal from parietal

1) If someone spills very hot coffee (200 0F) on their skin, they will likely perceive much pain. Which of the following receptor types is causing this sensation? A) nociceptors B) chemoreceptors C) mechanoreceptors D) thermoreceptors

A) nociceptors

16) The parasympathetic fibers of the ________ nerves innervate smooth muscles of the eye that cause the lenses to bulge to accommodate close vision. A) oculomotor (III) B) abducens (VI) C) optic (II) D) trochlear (IV)

A) oculomotor (III)

31) Which of the following would you NOT find in normal cerebrospinal fluid? A) red blood cells B) glucose C) protein D) potassium

A) red blood cells

B) Relative refractory period

An exceptionally strong stimulus can trigger a response A) Action potential B) Relative refractory period C) Repolarization D) Absolute refractory period E) Depolarization

Which of the following is true regarding a response to an excitatory event which might occur soon after the initial stimulus indicated in the graph?

An excitatory event may result in an action potential, but this will be less likely if the excitatory stimulus occurs during the response to the stimulus observed in the graph

Which statement(s) about the anatomical and functional divisions of the nervous systems are true?

Anatomical division includes the CNS (brain and spinal cord) and the PNS (nerves and ganglia).

Star-shaped neuroglia between neurons and blood vessels

Astrocyte

At which point of the illustrated action potential are the most gated Na+ channels open?

B

Bacteria in the blood can be prevented entry to the fluids surrounding the brain. Which cells are most responsible for preventing bacteria from entering the fluids of the brain? A) satellite cells B) astrocytes C) Schwann cells D) oligodendrocytes

B

If a signal from a sending neuron makes the receiving neuron more negative inside, A) the receiving neuron is more likely to generate an action potential. B) the receiving neuron is less likely to generate an action potential. C) the sending neuron becomes more positive inside. D) the sending neuron becomes more negative inside. E) the receiving neuron immediately generates an action potential.

B

Which lettered region in the figure is referred to as the soma?

B

Which of the cell types shown helps determine capillary permeability?

B

Biogenic amine neurotransmitters include all but: A) norepinephrine B) acetylcholine C) dopamine D) serotonin

B) acetylcholine

23) Which of the following describes the nervous system integrative function? A) responds to stimuli by gland secretion or muscle contraction B) analyzes sensory information, stores information, makes decisions C) senses changes in the environment

B) analyzes sensory information, stores information, makes decisions

7) Bacteria in the blood can be prevented entry to the fluids surrounding the brain. Which cells are most responsible for preventing bacteria from entering the fluids of the brain? A) satellite cells B) astrocytes C) Schwann cells D) oligodendrocytes

B) astrocytes

29) Which of the following adrenergic neurotransmitter receptors plays the major role in heart activity? A) alpha 1 B) beta 1 C) beta 2 D) beta 3

B) beta 1

27) All of the following are structures of the limbic system EXCEPT the ________. A) cingulate gyrus B) caudate nucleus C) amygdaloid body D) hippocampus

B) caudate nucleus

5) The parasympathetic ganglion that serves the eye is the ________. A) submandibular ganglion B) ciliary ganglion C) pterygopalatine ganglion D) otic ganglion

B) ciliary ganglion

53) The cells of the retina in which action potentials are generated are the ________. A) amacrine cells B) ganglion cells C) rods and cones D) bipolar cells

B) ganglion cells

28) Ordinarily, it is not possible to transplant tissues from one person to another, yet corneas can be transplanted without tissue rejection. This is because the cornea ________. A) has no nerve supply B) has no blood supply C) does not contain connective tissue D) is not a living tissue

B) has no blood supply

24) The ability to clearly see objects at a distance but not close up is properly called ________. A) hypopia B) hyperopia C) myopia D) presbyopia

B) hyperopia

51) What connects the hypothalamus to the pituitary gland? A) mammillary bodies B) infundibulum C) arbor vitae D) optic chiasma

B) infundibulum

27) Which of the following is not a function of the autonomic nervous system? A) innervation of smooth muscle of the digestive tract B) innervation of skeletal muscle C) innervation of cardiac muscle D) innervation of glands

B) innervation of skeletal muscle

56) The tarsal plate of the eyelid ________. A) is composed of connective tissue surrounding a thin cartilage plate B) is connected to the levator palpebrae C) is connected to the superior rectus muscle D) assists in the act of winking

B) is connected to the levator palpebrae

34) Which of the following correctly describes a graded potential? A) it has a depolarization, repolarization and hyperpolarization phase B) it can have amplitudes of various sizes C) it is initiated by voltage changes in the membrane D) it travels long distances

B) it can have amplitudes of various sizes

16) Neural tracts that convey information to the brain concerning temperature and pain would be ________. A) reticulospinal B) lateral spinothalamic C) ventral (anterior) spinothalamic D) posterior spinothalamic

B) lateral spinothalamic

25) Which part of the cerebral cortex is involved in intellect, cognition, recall, and personality? A) limbic association area B) prefrontal cortex (anterior association area) C) posterior association area D) combined primary somatosensory cortex and somatosensory association cortex

B) prefrontal cortex (anterior association area)

42) Which of the following is (are) involved with motor activity (either initiation or coordination)? A) Wernicke's area B) red nuclei C) gustatory cortex D) postcentral gyrus

B) red nuclei

29) The oval window is connected directly to which passageway? A) pharyngotympanic tube B) scala vestibuli C) external acoustic meatus D) scala tympani

B) scala vestibuli

21) Once a sympathetic preganglionic axon reaches a trunk ganglion, it can do all of the following EXCEPT? A) ascend or descend the trunk to synapse in another trunk ganglion B) synapse with a parasympathetic neuron in the same trunk ganglion C) pass through the trunk ganglion without synapsing with another neuron D) synapse with a ganglionic neuron in the same trunk ganglion

B) synapse with a parasympathetic neuron in the same trunk ganglion

4) The primary auditory cortex is located in the ________. A) frontal lobe B) temporal lobe C) parietal lobe D) prefrontal lobe

B) temporal lobe

32) REM sleep is associated with ________. A) decreased oxygen use, especially in the cerebral cortex B) temporary skeletal muscle inhibition except for ocular muscles and diaphragm C) decreased vital signs, such as heart rate and blood pressure D) decreased activity of the brain, especially the cerebral cortex

B) temporary skeletal muscle inhibition except for ocular muscles and diaphragm

42) A second nerve impulse cannot be generated until ________. A) all sodium gates are closed B) the membrane potential has been reestablished C) the Na ions have been pumped back into the cell D) proteins have been resynthesized

B) the membrane potential has been reestablished

63) Which statement about sound localization is not true? A) It requires input from both ears. B) It is difficult to discriminate sound sources in the midline. C) It requires processing at the cortical level. D) It uses time differences between sound reaching the two ears.

C) It requires processing at the cortical level.

24) The period after an initial stimulus when a neuron is not sensitive to another stimulus is the ________. A) repolarization B) resting period C) absolute refractory period D) depolarization

C) absolute refractory period

3) The somatic and autonomic nervous systems differ in all of the following EXCEPT ________. A) their effectors B) target organ responses to their neurotransmitters C) regulation of activity by higher brain centers D) their efferent pathways and ganglia

C) regulation of activity by higher brain centers

25) Which is a uniquely sympathetic function? A) regulation of pupil size B) regulation of respiratory rate C) regulation of body temperature D) regulation of cardiac rate

C) regulation of body temperature

62) The light of dawn and the buzz of an alarm clock lead to wakefulness through the ________. A) limbic system B) basal nuclei C) reticular activating system D) cerebellar peduncles

C) reticular activating system

45) Which of the following describes the excitatory postsynaptic potential? A) moves membrane potential away from threshold B) opens K+ or Cl- channels C) short distance depolarization D) short distance hyperpolarization

C) short distance depolarization

13) Dermatome maps are useful to clinicians because ________. A) they show the routes of motor nerves B) they outline the location of the numerous nerve plexus C) they can help pinpoint the location of spinal injury D) they show doctors how to avoid striking spinal nerves during surgery

C) they can help pinpoint the location of spinal injury

19) The posterior side of the thigh, leg, and foot is served by the ________ nerve. A) femoral B) obturator C) tibial D) common fibular

C) tibial

26) Striking the "funny bone" is actually stimulation of (or injury to) the ________. A) median nerve B) radial nerve C) ulnar nerve D) sciatic nerve

C) ulnar nerve

37) The area of the cortex that is responsible for sensing a full bladder and the feeling that your lungs will burst when you hold your breath too long is the ________. A) gustatory cortex B) vestibular cortex C) visceral sensory area D) olfactory cortex

C) visceral sensory area

36) Immediately after an action potential has peaked, which of the following channels will open? A) voltage gated calcium channels B) chemically gated chloride channels C) voltage gated potassium channels D) voltage gated sodium channels

C) voltage gated potassium channels

When a neurotransmitter like acetylcholine is acting in an excitatory manner which of the following is likely a result of the acetylcholine acting on the post synaptic cell?

Chemically gated sodium channels will open.

As you start working out, you notice that your heart rate and breathing rate start to increase. Which division of your nervous system is generating this response? Be as specific as possible. A) the somatic nervous system B) the afferent division of the nervous system C) the parasympathetic division of the autonomic nervous system D) the sympathetic division of the autonomic nervous system

D

10) A bit of dust blows into and touches the cornea of the eye. Which of the following is likely to happen? A) Stimulation of the facial nerve (VII) will be perceived as pain. B) Stimulation of the optic nerve (II) will cause tears to flow from the lacrimal gland. C) Nothing, because there is no sensory information sent from the cornea. D) Stimulation of the ophthalmic division of the trigeminal nerve (V) will cause blinking.

D) Stimulation of the ophthalmic division of the trigeminal nerve (V) will cause blinking.

1) Nutrients are delivered and waste products are carried away from the cells of the posterior segment of the eye by blood vessels. However, the cells in the cornea and lens (in the anterior segment) are avascular. Which is the best explanation for how these cells are maintained? A) The nutrients and waste products of the anterior segment diffuse into and through the vitreous humor of the posterior segment. B) The metabolic activity of these cells is very low. They produce little waste and need few nutrients. C) These cells, like the cells of the corneal layer of the integument are not living cells. D) The aqueous humor is continuously replenished and flows from the ciliary process to drain in the scleral venous sinus.

D) The aqueous humor is continuously replenished and flows from the ciliary process to drain in the scleral venous sinus.

9) An emergency medical technician is examining a trauma victim by shining a pen light into her patient's eye. She records the reactivity of the patents pupils as they constrict when stimulated by the light. This test supports which of the following? A) The patient has suffered brain damage. B) The patient has function of the trochlear nerve (IV). C) The patient has lost function of the optic nerve (II). D) The patient has function of the oculomotor nerve (III).

D) The patient has function of the oculomotor nerve (III).

25) Which of the following is not characteristic of neurons? A) They have an exceptionally high metabolic rate. B) They conduct impulses. C) They have extreme longevity. D) They are mitotic.

D) They are mitotic.

7) Over 90% of all parasympathetic fibers are derived from cranial nerves ________. A) VII (facial) B) XII (hypoglossal) C) V (trigeminal) D) X (vagus)

D) X (vagus)

47) If the ventral root of a spinal nerve were cut, what would be the result in the tissue or region that nerve supplies? A) a complete loss of sensation and movement B) loss of neither sensation nor movement but only of autonomic control C) complete loss of sensation D) a complete loss of voluntary movement

D) a complete loss of voluntary movement

27) Autonomic dysreflexia ________. A) usually precedes spinal shock B) results from over-excitatory input from the cortex C) is also known as autonomic areflexia D) involves uncontrolled activation of autonomic neurons

D) involves uncontrolled activation of autonomic neurons

6) The elasticity of the lens decreases with age. This leads to which of the following? A) less light getting to the retina and diminished visual acuity B) lowered accommodation of the pupillary reflex and blurry vision C) a clouding of the lenses know as a cataract D) less accommodation of the lenses and difficulty focusing on nearby objects

D) less accommodation of the lenses and difficulty focusing on nearby objects

42) A fracture of the ethmoid bone could result in damage to which cranial nerve? A) vagus B) glossopharyngeal C) accessory D) olfactory

D) olfactory

what structure is A

Dendrites A indicates the dendrites of the neuron

A The defects in nerve transmission associated with MS are caused by a loss of myelin within the CNS.

Destruction of which of the neuroglial cell types leads to the disease multiple sclerosis (MS)? A B D F

One or a few inputs ultimately influence large numbers of neurons

Diverging

When calcium ions enter the synaptic terminal A) the inside of the receiving neuron becomes more negative. B) they cause an action potential in the sending neuron. C) the inside of the receiving neuron becomes more positive. D) neurotransmitter molecules are quickly removed from the synaptic cleft. E) they cause vesicles containing neurotransmitter molecules to fuse to the plasma membrane of the sending neuron.

E

Which area would contain an abundance of vesicles containing neurotransmitter?

E

Which of the neuroglial cell types shown are found in the peripheral nervous system (PNS)

E The cell shown in E wraps and insulates the soma of neurons within ganglia in the PNS.

Since at any moment a neuron is likely to have thousands of neurons releasing neurotransmitters at its surface, how is neuronal activity (to fire or not to fire) determined?

Each neuron's axon hillock keeps a "running account" of all signals it receives via temporal and spatial summation.

Structure capable of responding to motor impulse

Effector

B. movement and circulation of cerebrospinal fluid

Ependymal cells line many open cavities in the central nervous system (CNS). Ependymal cells have cilia on the side of the cell that face these openings. What is the most likely function of these ciliated cells? ANSWER: A. create myelin sheaths for CNS cells B. movement and circulation of cerebrospinal fluid C. act as macrophage cells to destroy microorganisms or neuronal debris D. exchange of nutrients between the circulatory system and neurons

Some synapses are inhibitory, and magnify the impulse.

False

A) Multipolar (CNS)

In a reflex arc, which neuron has its cell body inside the spinal cord?

Loss of function in the enzyme acetylcholinesterase would result in which of the following?

Inability to destroy and remove acetylcholine from synaptic cleft

B) Postsynaptic membrane

Match: Postsynaptic membrane

D) Synaptic cleft

Match: Synaptic cleft

C) Synaptic vesicles

Match: Synaptic vesicles

Multipolar, Bipolar, or Unipolar? Motor neuron

Multipolar

The Na+-K+ pump actively transports both sodium and potassium ions across the membrane to compensate for their constant leakage. In which direction is each ion pumped?

Na+ is pumped out of the cell and K+ is pumped into the cell.

Tremor at rest, shuffling gait, stooped posture, and expressionless face are characteristics of ________.

Parkinson's disease

The action potential is caused by permeability changes in the plasma membrane.

T

Unipolar neurons have axons structurally divided into peripheral and central processes.

T

D) brain and spinal cord

The term central nervous system refers to the ________. A) the somatic nerves B) sensory (afferent) nerves C) the spinal nerves D) brain and spinal cord

Possesses a single process from the cell body

Unipolar Neuron

Na+ ions move into the axon; K+ ions move out

What best describes the direction followed by the specified ions when their voltage-gated channels open?

the voltage measured across the axon membrane at a specific point as an action potential travels past.

What change in a neuron is being measured in the graph?

Na+ channels are inactivating, and K+ channels are opening.

What is happening to voltage-gated channels at this point in the action potential?

dendrites A indicates the dendrites of the neuron.

What is the structure at A?

Threshold Stimulus

any stimulus at or above threshold level will result in action potential

Bipolar neurons are commonly ________. a) found in ganglia b) found in the retina of the eye c) called neuroglial cells d) motor neurons

b

During depolarization, which gradient(s) move(s) Na+ into the cell?

both the electrical and chemical gradients

Broca's area ________________.

is considered a motor speech area

If the resting membrane potential is - 70 mV and the threshold is -55 mV, a membrane potential of -60 mV will

make it easier to produce an action potential

A molecule that carries information across a synaptic cleft is a

neurotransmitter

Which of the following is not a function of astrocytes?

provide the defense for the CNS

A postsynaptic potential is a graded potential that is the result of a neurotransmitter released into the synapse between two neurons.

T

Axon diameter and degree of myelination determine nerve impulse conduction velocity.

T

Cell bodies of sensory neurons may be located in ganglia lying outside the central nervous system.

T

During depolarization, the inside of the neuron's membrane becomes less negative.

T

Enkephalins and endorphins are peptides that act like morphine

T

Neurons in the CNS are organized into functional groups.

T

Synaptic transmission of a nerve impulse involves

a presynaptic neuron with a neurotransmitter a postsynaptic neuron with receptors for the neurotransmitter an inactivator for the neurotransmitter a synapse (space to be crossed)

The knee-jerk reflex is an example of a __________.

stretch reflex

Which reflex requires gamma motor neurons to set the length of the muscle?

stretch reflex

Neurons are ____, whiIe neuroglial cells are _____.

structural and functional units of the nervous system; cells that provide frameworks and protection for the neurons

A shallow groove on the surface of the cortex is called a __________.

sulcus

The peripheral nervous system consists of

the peripheral nerves

peripheral nervous system

the portion of the nervous system outside the CNS; communication lines between the CNS and the rest of the body

If a post synaptic neuron is stimulated to threshold by spatial summation this implies that

the postsynaptic cells has many synapses with many presynaptic neurons

spastic paralysis suggests involvement of the _________.

upper motor neurons

Which cranial nerve innervates most of the visceral organs?

vagus

What is the function of microglia?

- Structural support - Phagocytosis of microorganisms and damaged tissues

What is the value for the resting membrane potential for most neurons?

-70mV

A graded potential

-decreases with distance from the point of stimulation -spreads passively because of local currents -may involve either depolarization or hyperpolarization

Summation

-is caused by combining two or more local potentials -occurs at the trigger zone of the postsynaptic neuron -results in an action potential if it reaches the threshold potential -can occur when two action potentials arrive in close succession at a single presynaptic terminal

An inhibitory presynaptic neuron can affect a postsynaptic neuron by

-producing an IPSP in the postsynaptic neuron -hyperpolarizing the plasma membrane of the postsynaptic neuron -causing K+ to diffuse out of the postsynaptic neuron -causing Cl- to diffuse into the postsynaptic neuron

Match the names of the supporting cells with the appropriate description 1 myelinates nerve fibers in CNS 2 lines brain cavities 3 myelinates nerve fibers in the PNS 4 CNS phagocytes 5 helps regulate the ionic composition of CNS extracellular fluid

1 Oligodendrocyte 2 Ependymal cell 3 microglia 4 Schwann Cell 5 astrocyte

Match the neuron with the description of its structure: 1 multipolar neurons 2 bipolar neurons 3 unipolar neurons

1 many dendrites and one axon 2 one axon and one dendrites 3 one process and one dendrite into an axon and a dendrite

Identify the neuronal circuits described by choosing the correct response 1 Impulses continue around and around the circuit until one neuron stops firing 2 One or a few imputs ultimately influence large numbers of neurons 3 Many neurons influence a few neurons 4 May be involved in exacting types of mental activity

1 reverberating 2 diverging 3 converging 4 parallel after discharge

A) E B) B C) C D) D 1. Innervates the superior oblique muscle. 2. Longest cranial nerve. 3. Damage to this nerve would cause dizziness, nausea, and loss of balance. 4. Involved in movement of the digestive tract. 5. Damage to this nerve would cause difficulty in speech and swallowing, but no effect on visceral organs. 6. Damage to this nerve would keep the eye from rotating inferolaterally.

1) B 2) D 3) C 4) D 5) A 6) B

A) B B) E C) A D) D E) C 1) Ganglion cells. 2) Bipolar cells. 3) Horizontal cell. 4) Amacrine cell. 5) Rod.

1) B 2) E 3) C 4) D 5) A

(1.) What is myelin? (2.) How does the myelination process differ in the CNS and PNS?

1. Myelin is a whitish, fatty, phospholipid-insulating material (essentially the wrapped plasma membranes of oligodendrocytes or Schwann cells). 2. CNS myelin sheaths are formed by flap like extensions of oligodendrocytes and lack an outer collar of perinuclear cytoplasm. Each oligodendrocyte can help to myelinate several fibers. PNS myelin is formed by Schwann cells; the wrapping of each Schwann cell forms the internode region. The sheaths have an outer collar of perinuclear cytoplasm, and the fibers they protect are capable of regeneration.

(1.) Describe the composition and function of the cell body. (2.) How are axons and dendrites alike? In what ways (structurally and functionally) do they differ?

1. The cell body is the biosynthetic and metabolic center of a neuron. It contains the usual organelles, but lacks centrioles. 2. Dendrites and axons are alike in that they both function to carry electrical current. Dendrites differ in that they are short, transmit toward the cell body, and function as receptor sites. Axons are typically long,myelinated, and transmit away from the cell body. Only axons can generate action potentials, the long-distance signals.

Describe the events that must occur to generate an AP. Relate the sequence of changes in permeability to changes in the ion channels and explain why the AP is an all-or-none phenomenon.

1. The generation of an action potential involves changes in the state of ion channels in response to changes in membrane potential, which leads to: (1) an increase in sodium permeability and reversal of the membrane potential; (2) a decrease in sodium permeability; and (3) an increase in potassium permeability and repolarization. 2. The all-or-none phenomenon means that the local depolarizing current must reach a critical threshold point before membrane channels will respond and when a response occurs, it will lead to the conduction of an action potential along the entire length of the axon.

A) A B) C C) B D) E E) D 11) Acts as a reflexively activated diaphragm to vary pupil size. 12) The only tissue in the body that can be transplanted from one person to another with little or no rejection. 13) Controls lens shape. 14) Viscous liquid that holds the retina firmly against the pigmented layer. 15) Fluid blockages cause glaucoma.

11) B 12) C 13) A 14) D 15) E

A) D B) A C) C D) B E) E 11) Site of somatic motor neuron cell bodies. 12) Consists entirely of interneuron cell bodies. 13) Site of sensory neuron cell bodies. 14) Gray commissure. 15) Site of autonomic motor neuron cell bodies. 16) Area that encloses the central canal. 17) Unipolar neurons are found here.

11) C 12) D 13) E 14) B 15) A 16) B 17) E

A) Olfactory B) Accessory C) Abducens D) Vagus E) Vestibulocochlear 12) Rootlets arise on the spinal cord, enter the skull through the foramen magnum and exit the skull through the jugular foramen. 13) Receptors located in epithelium of the nasal cavity. 14) Serves the senses of hearing and equilibrium. 15) Helps to regulate blood pressure and digestion. 16) Turns the eyeball laterally.

12) B 13) A 14) E 15) D 16) C

A) Norepinephrine (NE) B) Acetylcholine (ACh) 12) Secreted by preganglionic sympathetic fibers. 13) Secreted by preganglionic parasympathetic fibers. 14) Secreted by postganglionic sympathetic fibers to sweat glands. 15) Secreted by postganglionic parasympathetic fibers. 16) Secreted by most postganglionic sympathetic fibers.

12) B 13) B 14) B 15) B 16) A

A) Primary (somatic) motor cortex B) Hypothalamus C) Thalamus D) Prefrontal cortex 26) A major relay station for sensory information ascending to the cerebral cortex. Contains many specialized nuclei. 27) This brain area associates experiences necessary for the production of abstract ideas, judgment, and conscience. 28) The axons from this area form the major pyramidal (corticospinal) tracts. 29) This area is the main visceral control center of the body.

26) C 27) D 28) A 29) B

A) Segmental level B) Projection level C) Precommand level 27) Controls the outputs of the cortex and regulates motor activity. 28) Central pattern generators. 29) Intermediate relay for incoming and outgoing neurons. 30) The cerebellum and basal nuclei. 31) Includes cortical and brain stem motor areas. 32) The neural machinery of the spinal cord, including spinal cord circuits.

27) C 28) A 29) B 30) C 31) B 32) A

A) B B) A C) C 33) Contain autonomic nerve fibers. 34) Supplies the posterior body trunk. 35) Supplies all of the body except the posterior. 36) All but T2-T12 branch and form nerve plexuses. 37) Supplies each muscle with fibers from more

33) C 34) B 35) A 36) A 37) A

A) Sympathetic B) Parasympathetic 5) Short preganglionic, long postganglionic fibers. 6) Collateral ganglia. 7) Active after you have eaten a meal. 8) Decreases heart rate. 9) Maintenance functions. 10) Stimulates ciliary muscles of the eye, which makes the lens bulge for close vision. 11) Increases blood pressure.

5) A 6) A 7) B 8) B 9) B 10) B 11) A

The small space between the sending neuron and the receiving neuron is the A) synaptic cleft. B) synaptic terminal. C) neurotransmitter. D) calcium channel. E) vesicle.

A

These cells in the CNS have cilia that move in order to circulate cerebrospinal fluid ________. A) ependymal cells B) Schwann cells C) oligodendrocytes D) astrocytes

A

When a neurotransmitter like GABA is acting in an inhibitory manner which of the following is likely a result of the GABA acting on the post synaptic cell? A) hyperpolarization of the postsynaptic cell membrane B) influx of positively charged ions into the postsynaptic cell C) depolarization of the postsynaptic cell membrane D) opening of chemically gated sodium channels

A

Destruction of which of the neuroglial cell types leads to the disease multiple sclerosis (MS)

A The defects in nerve transmission associated with MS are caused by a loss of myelin within the CNS.

Which of the neuroglial cell types shown form myelin sheaths within the CNS?

A These cells form myelin sheaths within the white matter of the CNS

Which of the following does NOT describe conditions that occur during an action potential? A) Na+ is used to repolarize the membrane. B) Na+ enters the cell through voltage-gated channels, causing a reversal of the resting membrane potential. C) The Na+/K+ pump reestablishes resting concentration gradients. D) After initial depolarization, K+ now has both an electrical and a chemical gradient drawing it out of the cell.

A This is false. Initially, K+ leaves the cell through voltage-gated channels. As K+ leaves the cell, the inside of the cell becomes more negative, repolarizing the cell.

D) association neuron

A neuron that has as its primary function the job of connecting other neurons is called a(n) ________. A) afferent neuron B) efferent neuron C) glial cell D) association neuron

Unipolar Neuron

A single process that divides into two branches and functions as an axon

Which of the following will occur when an excitatory postsynaptic potential (EPSP) is being generated on the dendritic membrane?

A single type of channel will open, permitting simultaneous flow of sodium and potassium.

55) The ________ includes the thalamus, hypothalamus, and epithalamus. A) diencephalon B) basal nuclei C) midbrain D) brain stem

A) diencephalon

19) These cells in the CNS have cilia that move in order to circulate cerebrospinal fluid ________. A) ependymal cells B) astrocytes C) oligodendrocytes D) Schwann cells

A) ependymal cells

4) Tactile sensation is a combination of touch, pressure, stretch and vibration. Which of the following is most likely the receptor type that senses tactile stimulation? A) mechanoreceptors B) proprioceptors C) nociceptors D) thermoreceptors

A) mechanoreceptors

3) The brain stem consists of the ________. A) midbrain, medulla, and pons B) pons, medulla, cerebellum, and midbrain C) midbrain only D) cerebrum, pons, midbrain, and medulla

A) midbrain, medulla, and pons

A) Action potential

Also called a nerve impulse transmitted by axons A) Action potential B) Relative refractory period C) Repolarization D) Absolute refractory period E) Depolarization

Which of the following occurred in the presence of tetrodotoxin

An action potential was always seen at R1.

Nerve-fiber arising from a slight elevation of the cell body that conducts an action potential (impulse) away from the cell body

Axon

The conducting region of the neuron is the

Axon

Unmyelinated Axon

Axon without myelin sheath around it

22) Bitter taste is elicited by ________. A) acids B) alkaloids C) hydrogen ions D) metal ions

B) alkaloids

Strong stimuli cause the amplitude of action potentials generated to increase

False

Which areas of this neuron would be classified as receptive regions?

Both A and B The dendrites and soma of the cell receive signals from other neurons

A) exocytosis The influx of Ca2+ triggers the release of neurotransmitters stored in synaptic vesicles (B) by exocytosis.

By which method does the structure at B release neurotransmitter? A) exocytosis B) facilitated diffusion C) passive diffusion D) active transport

36) Which nerve is compressed in carpal tunnel syndrome? A) ulnar B) radial C) median D) axillary

C) median

6) Cardiovascular effects of the sympathetic division include all of the following EXCEPT ________. A) weak dilation of the blood vessels of skeletal muscles during exercise B) increase of heart rate and force C) dilation of the blood vessels serving the skin and digestive viscera D) constriction of most blood vessels

C) dilation of the blood vessels serving the skin and digestive viscera

8) Meningitis can be caused by infection of the central nervous system by bacteria. Which cells would be most responsible for removing the infection? A) Schwann cells B) oligodendrocytes C) microglia D) satellite cells

C) microglia

36) Which of the following drug classes would be useful for smoking cessation products? A) sympathomimetic agents B) acetylcholinesterase inhibitors C) nicotinic agents D) parasympathomimetic agents (muscarinic agents)

C) nicotinic agents

47) Which brain waves are uncommon for awake adults, but are common for children? A) delta B) alpha C) theta D) beta

C) theta

37) Which of the following taste sensations is incorrectly matched to the chemicals that produce it? A) salty-metal ions B) sweet-organic substances such as sugar and some lead salts C) umami-triglycerides and fatty acids D) bitter-alkaloids E) sour-acids

C) umami-triglycerides and fatty acids

Many neurons influence a few neurons

Converging

What is the shape of ependymal cell?

Cuboidal cells lining cavities of the brain and spinal cord.

Meningitis can be caused by infection of the central nervous system by bacteria. Which cells would be most responsible for removing the infection? A) satellite cells B) oligodendrocytes C) Schwann cells D) microglia

D

Which step of nerve or muscle firing would be directly affected by a change in extracellular K+ ? A) Depolarization B) Sodium channel opening C) Action potential D) Repolarization E) Sodium channel closing

D

25) Seventy percent of all sensory receptors are located in the ________. A) skin B) nose C) ears D) eye

D) eye

16) When a neurotransmitter like GABA is acting in an inhibitory manner which of the following is likely a result of the GABA acting on the post synaptic cell? A) influx of positively charged ions into the postsynaptic cell B) opening of chemically gated sodium channels C) depolarization of the postsynaptic cell membrane D) hyperpolarization of the postsynaptic cell membrane

D) hyperpolarization of the postsynaptic cell membrane

18) Which of the following is NOT a plexus of the vagus (X) nerve? A) pulmonary B) esophageal C) cardiac D) inferior hypogastric

D) inferior hypogastric

26) Raynaud's disease ________. A) is frequently life threatening B) is induced by heat stress C) occurs primarily in association with injury to the spinal cord D) is characterized by exaggerated vasoconstriction in the extremities

D) is characterized by exaggerated vasoconstriction in the extremities

7) Photoreceptors are modified neurons. However, they differ from most neurons in the fact that ________. A) photoreceptors do not have fluctuations in membrane potential B) photoreceptors do not release neurotransmitters C) photoreceptors are sensitive to energy changes in the external environment D) photoreceptors will hyperpolarize when stimulated

D) photoreceptors will hyperpolarize when stimulated

20) Starting at the spinal cord, the subdivisions of the brachial plexus are (in order) ________. A) divisions, roots, trunks, and cords B) trunks, divisions, cords, and roots C) roots, divisions, cords, and trunks D) roots, trunks, divisions, and cords

D) roots, trunks, divisions, and cords

10) Which of these effectors is NOT directly controlled by the autonomic nervous system? A) most glands B) smooth muscle C) cardiac muscle D) skeletal muscle

D) skeletal muscle

38) Another name for the primary visual cortex is ________. A) prefrontal cortex B) prestriate cortex C) collicular cortex D) striate cortex

D) striate cortex

12) A shallow groove on the surface of the cortex is called a ________. A) gyrus B) furrow C) fissure D) sulcus

D) sulcus

1) Which brain nucleus is the body's "biological clock"? A) lentiform nucleus B) dorsomedial nucleus C) subthalamic nucleus D) suprachiasmatic nucleus

D) suprachiasmatic nucleus

20) Spastic paralysis suggests involvement of the ________. A) spinal nerve roots B) lower motor neurons C) neuromuscular junction D) upper motor neurons

D) upper motor neurons

What factors appear to guide the outgrowth of an axon and its ability to make the "correct" synaptic contacts.

Development of the axon is due to the chemical signals neurotropin and NGF (nerve growth factor) that interact with receptors on the developing axon to support and direct its growth.

The effect of acetylcholine can be stimulating or inhibiting. Which of the following gives the best explanation for why this is so?

Different post synaptic cells will have different receptors.

Briefly describe the three stages of neuron development.

During the first developmental stage, neurons proliferate; during the second stage, neurons migrate to proper position; during the third stage, neurons differentiate.

Cells that line cavities of the brain and secrete cerebrospinal fluid

Ependymal cell

Lines brain cavity

Ependymal cell

What is the function of an oligodendrocyte?

Form myelin sheaths in the brain and spinal cord

Distinguish between serial and parallel processing.

In serial processing, the pathway is constant and occurs through a definite sequence of neurons; the response is predictable and stereotyped. In parallel processing, impulses reach the final CNS target by multiple pathways. Parallel processing allows for a variety of responses.

C Graded potentials originating in the dendrites and cell body are integrated (summated) at the axon hillock.

In which area of the neuron is an action potential initially generated?

What changes occur to voltage-gated Na+ and K+ channels at the peak of depolarization?

Inactivation gates of voltage-gated Na+ channels close, while activation gates of voltage-gated K+ channels open.

Transmits impulse from sensory to motor neuron within central nervous system

Interneuron

Signals generated at a chemical synapse are said to move only in the direction of the presynaptic cell to the postsynaptic cell. Which of the following statements regarding the mechanisms determining this one way transmission is INCORRECT?

Ions diffusing out of the presynaptic cell can enter the postsynaptic cell, but cannot reenter the presynaptic cell.

The resting membrane potential results when the tendency for ______ to diffuse out of the cell is balanced by its attraction to opposite charges inside the cell.

K+

During repolarization of the plasma membrane

K+ diffuses out of the cell

Transmits impulse out of the brain or spinal cord to effectors (muscles and glands)

Motor (efferent) neuron

B) somatic nervous system

Nerve impulses leading to the skeletal muscle carry information to direct movement. The nerve fibers sending these signals will most likely belong to which division of the nervous system? A) sensory (afferent) division B) somatic nervous system C) sympathetic division D) parasympathetic division

A _____________ neuron is the most common structural neuron in the brain and spinal cord a. Multipolar b. Tripolar c. Bipolar d. Unipolar

Multipolar

What structural classification describes this neuron?

Multipolar The neuron shown has a many processes (axon and dendrites) that emerge from the cell body. Such neurons typically function as motor neurons or interneurons

Substance of Schwann cell composed of lipoprotein that insulates axons and increases impulse speed

Myelin

Sheath of Schwann cell containing cytoplasm and nucleus that encloses myelin

Neurilemma

Network of threadlike structures within cell body and extending into axon

Neurofibrils

Which of these materials or structures would be found in greatest amounts or numbers at E? ( the tail)

Neurotransmitters are released by secretion from the ends of axonal terminals.

Corresponds to rough endoplasmic reticulum

Nissl bodies

Which of the following is a way in which the somatic and autonomic nervous systems are similar?

None of the listed responses are correct

D) The inside surface of the plasma membrane is much more negatively charged than the outside surface. The inside surface of the plasma membrane accumulates more negative charge because of the presence of Na+ and K+ gradients and the selective permeability of the membrane to Na+ and K+.

On average, the resting membrane potential is -70 mV. What does the sign and magnitude of this value tell you? A) The outside surface of the plasma membrane is much more negatively charged than the inside surface. B) There is no electrical potential difference between the inside and the outside surfaces of the plasma membrane. C) The inside surface of the plasma membrane is much more positively charged than the outside surface. D) The inside surface of the plasma membrane is much more negatively charged than the outside surface.

Bipolar Neuron

One axon and one dendrite

B) the myelin sheath

Saltatory conduction is made possible by ________. A) erratic transmission of nerve impulses B) the myelin sheath C) diphasic impulses D) large nerve fibers

Myelinates nerve fibers in the PNS

Schwann cell

myelinates nerve fibers in the PNS

Schwann cell

what is the type of cell that carry out myelination of the Peripheral nervous system

Schwann cell

After axonal injury, regeneration in peripheral nerves is guided by ________.

Schwann cells

What is the shape of an oligodendrocyte?

Shaped like astrocytes, but with fewer cellular processes

What is the shape of microglia?

Small cells with slender cellular processes

C) Leak channels Leak channels for Na+ and K+ are ubiquitous, and they allow for the diffusion of these ions across plasma membranes.

Sodium and potassium ions can diffuse across the plasma membranes of all cells because of the presence of what type of channel? A) Ligand-gated channels B) Voltage-gated channels C) Leak channels D) Sodium-potassium ATPases

Which of the following occurs during depolarization

Sodium flows into the cell.

Which of the following is true about the movement of ions across excitable living membranes?

Sodium gates in the membrane can open in response to electrical potential changes.

Why does the threshold increase when the interval between the stimuli decreases?

Some sodium channels have been inactivated and cannot be reopened immediately

What is the shape of an astrocyte?

Star-shapes cells contacting neurons and blood vessles

D) reuptake of the neurotransmitter by transport into the postsynaptic cell Neurotransmitters are generally not transported into the postsynaptic cell upon which they exert their effects.

The box labeled D illustrates three mechanisms by which the effects of a neurotransmitter may be terminated. Which of the following mechanisms is NOT included in the figure? A) diffusion of the neurotransmitter out of the synaptic cleft B) reuptake of the neurotransmitter by transport into the presynaptic cell C) breakdown of the neurotransmitter within the synaptic cleft D) reuptake of the neurotransmitter by transport into the postsynaptic cell

The effects of neurotransmitter binding are very brief. Explain.

The effects of neurotransmitter binding are brief because the neurotransmitter is quickly removed by enzymatic degradation or re-uptake into the presynaptic axon. In the absence of a neurotransmitter on the receptor binding site, the chemically gated ion channels close, and the membrane potential returns to the resting state. This ensures discrete, limited responses.

E) Depolarization

The interior of the cell becomes less negative d/t an influx of sodium ions A) Action potential B) Relative refractory period C) Repolarization D) Absolute refractory period E) Depolarization

Which of the following occurs first in the generation of an action potential?

The membrane depolarizes

D) Absolute refractory period

The neuron cannot respond to a second stimulus, no matter how strong A) Action potential B) Relative refractory period C) Repolarization D) Absolute refractory period E) Depolarization

B) The presence of concentration gradients and leak channels The concentration gradient and the large number of K+ leak channels allow for rather robust K+ diffusion out of a cell. In contrast, the concentration gradient and the relatively few Na+ leak channels allow for much less Na+ diffusion into a cell.

The resting membrane potential depends on two factors that influence the magnitude and direction of Na+ and K+ diffusion across the plasma membrane. Identify these two factors. A) The presence of concentration gradients and voltage-gated channels B) The presence of concentration gradients and leak channels C) The presence of concentration gradients and Na+-K+ pumps D) The presence of a resting membrane potential and leak channels

What do Schwann cells do?

They are cells that wrap tightly around the axons of peripheral neurons; speed up neurotransmission

What do Satellite cells do?

They are flattened cells that surround cell bodies of neurons in ganglia; they support ganglia

Which of the following is not characteristic of neurons?

They are mitotic.

Which of the following is true of axons?

This is true; a neuron can have only one axon, but the axon may have occasional branches along its length.

Local anesthetics block voltage-gated Na+ channels, but they do not block mechanically gated ion channels. Sensory receptors for touch (and pressure) respond to physical deformation of the receptors, resulting in the opening of specific mechanically gated ion channels. Why does injection of a local anesthetic into a finger still cause a loss of the sensation of touch from the finger? Local anesthetics block voltage-gated Na+ channels, but they do not block mechanically gated ion channels. Sensory receptors for touch (and pressure) respond to physical deformation of the receptors, resulting in the opening of specific mechanically gated ion channels. Why does injection of a local anesthetic into a finger still cause a loss of the sensation of touch from the finger?

Touch stimulation of this sensory receptor will open the mechanically gated ion channels, but action potentials are still not initiated because propagation of an action potential requires the opening of voltage-gated Na+ channels

Interneuron

Transmits impulses between neurons in the brain and spinal cord

A stimulus traveling toward a synapse appears to open calcium ion channels at the presynaptic end, which in turn promotes fusion of synaptic vesicles to the axonal membrane.

True

Efferent nerve fibers may be described as motor nerve fibers

True

Which of the following pairings does NOT fit? (a) afferent neurons: sensory neurons (b) motor neurons: efferent neurons (c) multipolar neurons: peripheral nervous system sensory neurons (d) association neuron: functional classification

Unipolar neurons makeup the sensory neurons in the peripheral nervous system.

Which of the following membrane regions would have significant numbers of voltage-gated ion channels?

Voltage-gated Na+ and K+ channels allow for the triggering of an action potential at the axon hillock (C) and its propagation down the axon (D). Receptive regions of neurons are areas characterized by the presence of chemically gated ion channels and other neurotransmitter-binding receptors. ( they are located in the tail and the near the tail

Which of the following occurs during repolarization

Voltage-gated potassium channels open and some voltage-gated sodium channels inactivate. Potassium flows out of the cell.q

Explain both the anatomical and functional divisions of the nervous system. Include the subdivisions of each.

We only have one nervous system but it can be divided into two principal parts. A) Central nervous system (CNS)- Structure: brain and spinal cord Function: Integrative and control centers B) Peripheral nervous system (PNS)- Structure: cranial nerves and spinal nerves Function: communication lines between the CNS and the rest of the body PNS has two functional subdivisions: 1) Sensory Division: Structure: somatic and visceral sensory nerve fibers Function: Conducts impulses from receptors to the CNS 2) Motor Division: Structure: motor nerve fibers Function: conducts impulses from the CNS to effector (muscles and glands) Motor division has 2 main parts: 1) Somatic nervous system: Structure: somatic motor Function: conducts impulses from the CNS to skeletal muscle 2) Autonomic nervous system: Structure: visceral motor Function: conducts impulses from the CNS to cardiac muscles, smooth muscles, and glands. Autonomic nervous system has 2 functional subdivisions: 1) Sympathetic: Function: mobilizes body systems during activity 2) Parasympathetic: Function: Conserves energy and promotes housekeeping during rest

D) Na+-K+ ATPase Also known as the Na+-K+ pump, or simply the pump, this transporter moves three Na+ out of the cell and two K+ into the cell for every ATP it hydrolyzes. This pumping action prevents the Na+ and K+ gradients from running down as these ions passively move through leak channels.

What prevents the Na+ and K+ gradients from dissipating? A) Na+ and K+ leaks B) H+-K+ ATPase C) Na+ cotransporter D) Na+-K+ ATPase

multipolar The neuron shown has many processes (axon and dendrites) that emerge from the cell body. Such neurons typically function as motor neurons or interneurons.

What structural classification describes this neuron?

C) Initial segment The first part of the axon is known as the initial segment. The initial segment is adjacent to the tapered end of the cell body, known as the axon hillock.

Where do most action potentials originate? A) Nodes of Ranvier B) Cell body C) Initial segment D) Axon terminal

A) Multipolar (CNS)

Which is by far, the most common neuron type?

A. reverberating circuits

Which of the following circuit types is involved in the control of rhythmic activities such as the sleep-wake cycle, breathing, and certain motor activities (such as arm swinging when walking)? ANSWER: A. reverberating circuits B. diverging circuits C. converging circuits D. parallel after-discharge circuits

C. Na+ is used to repolarize the membrane.

Which of the following does NOT describe conditions that occur during an action potential? ANSWER: A. After initial depolarization, K+ now has both an electrical and a chemical gradient drawing it out of the cell. B. Na+ enters the cell through voltage-gated channels, causing a reversal of the resting membrane potential. C. Na+ is used to repolarize the membrane. D. The Na+/K+ pump reestablishes resting concentration gradients.

A. One EPSP fails to bring the cell to threshold.

Which of the following does NOT describe the process of summation? ANSWER: A. One EPSP fails to bring the cell to threshold. B. One EPSP and one IPSP cancel each other out. C. Two EPSPs occur close enough in time to bring the cell to threshold. D. Two EPSPs are generated at the same time by two separate synapses, bringing the cell to threshold.

The large commissure that connects the right and left sides of the brain is called the ________.

corpus callosum

B. Release hormones into the bloodstream to communicate with other cells in the body.

Which of the following is NOT one of the basic functions of the nervous system? ANSWER: A. Decode sensory information from the environment. B. Release hormones into the bloodstream to communicate with other cells in the body. C. Generate direct, electrical signals. D. Integrate sensory input for decision making.

D) They are mitotic.

Which of the following is not characteristic of neurons? A) They have an exceptionally high metabolic rate. B) They conduct impulses. C) They have extreme longevity. D) They are mitotic.

A. microglia

Which of the following types of glial cells monitors the health of neurons, and can transform into a special type of macrophage to protect endangered neurons? ANSWER: A. microglia B. ependymal cells C. astrocytes D. oligodendrocytes

Oligodendrocytes These cells form myelin sheaths within the white matter of the CNS.

Which of the neuroglial cell types shown form myelin sheaths within the CNS?

vesicles containing neurotransmitter Neurotransmitters are released by secretion from the ends of axonal terminals.

Which of these materials or structures would be found in the greatest amounts or numbers at E?

What is myelin?

White, fatty myelin sheath that protects and electrically insulates fibers. Increases the speed of transmission of nerve impulses.

A patient is admitted to the rehabilitation unit five days after having a stroke. The nurse assesses his muscle strength and determines that he has right-sided weakness. Based on this assessment data, what part of the brain was injured? a) There was damage to localized areas of the primary motor cortex in the left cerebral hemisphere. b) There was damage to localized areas of the premotor cortex in the right cerebral hemisphere. c) There was damage to localized areas of the prefrontal cortex in the right cerebral hemisphere. d) There was damage to localized areas of the primary motor cortex in the right cerebral hemisphere. e) There was damage to localized areas of the premotor cortex in the left cerebral hemisphere.

a

Broca's area ________. a) is considered a motor speech area b) serves the recognition of complex objects c) is usually found only in the right hemisphere d) controls voluntary movements of the eyes

a

Which of the following is the correct simple spinal reflex arc? a) receptor, afferent neuron, integration center, efferent neuron, effector b) effector, efferent neuron, integration center, afferent neuron, receptor c) effector, afferent neuron, integration center, efferent neuron, receptor d) receptor, efferent neuron, integration center, afferent neuron, effector

a

Which of the following statements about the cerebellum is NOT correct? a) The cerebellum generates conscious motor commands. b) The cerebellum plays a role maintaining your balance. c) The cerebellum has a cortex and homunculus, just like the motor cortex. d) Damage to the cerebellum could affect posture.

a

Which pair below is incorrect? a) cranial Nerve IV: sensory b) cranial nerve VI: eye movement c) optic nerve: sensory d) cranial nerve III: pupillary constriction e) cranial nerve III: oculomotor nerve

a

A nerve is

a bundle of axons

When the interval between the stimuli decreases,

a second action potential is generated until the interval reaches the absolute refractory period

The period after an initial stimulus when a neuron is not sensitive to another stimulus is the

absolute refractory period

The period after an initial stimulus when a neuron is not sensitive to another stimulus is the ________.

absolute refractory period

The cranial nerve with a cervical origin (spinal cord) is the ________.

accessory

Biogenic amine neurotransmitters include all but

acetylcholine

Which of the following is an excitatory neurotransmitter secreted by motor neurons innervating skeletal muscle?

acetylcholine

Biogenic amine neurotransmitter include all but

acetylcholine does include nonepinephrine, dopamine, and seratoning

in a chemical synapse

action potentials in the presynaptic terminal cause voltage-gated Ca+2 channels to open, neurotransmitters can cause ligand-gated Na+ channels to open, neurotransmitters can be broken down by enzymes, neurotransmitters can be taken up by the presynaptic terminal

A neuron that has as its primary function the job of connecting other neurons is called a(n)

afferent neuron

Where would you NOT find a cholinergeric nicotinic receptor?

all parasympathetic target organs

Action potentials are conducted more rapidly

along axons that have nodes of Ranvier

Bacteria in the blood can be prevented entry to the fluids surrounding the brain. Which cells are most responsible for preventing bacteria from entering the fluids of the brain?

astrocytes

Sub threshold Stimulus

an insufficient stimulus to reach an action potential

Which of the following describes the nervous system integrative function?

analyzes sensory information, stores information, makes decisions

The subarachnoid space lies between what two layers of meninges?

arachnoid and pia

neuron processes

armlike processes that extend from body

The concentration neurotransmitters in the fluid of the synaptic cleft must be tightly regulated for neurons to function properly. Which of the following cells is most responsible for aiding in this regulation?

astrocytes

The concentration of ions in the chemical environment surrounding the neurons must be tightly regulated for neurons to function properly. Which of the following cells is most responsible for this?

astrocytes

The part of the nervous system that controls smooth muscle, cardiac muscle, and glands is the

autonomic nervous system

Bacteria in the blood can be prevented entry to the fluids surrounding the brain. Which cells are most responsible for preventing bacteria from entering the fluids of the brain? a) satellite cells b) astrocytes c) Schwann cells d) oligodendrocytes

b

Which reflex is triggered when a stranger suddenly grasps your arm? a) stretch reflex b) crossed-extensor reflex c) tendon reflex d) plantar reflex

b

oligodendrocytes

branched cells that have processes that form myelin sheaths around CNS nerve fibers

nerves

bundles of neuron processes in PNS

A neuron that has as its primary function the job of connecting other neurons is called a(n) ________. a) afferent neuron b) glial cell c) association neuron d) efferent neuron

c

The small space between the sending neuron and the receiving neuron is the a) calcium channel. b) neurotransmitter. c) synaptic cleft. d) synaptic terminal. e) vesicle.

c

The term central nervous system refers to the ________. a) the spinal nerves b) sensory (afferent) nerves c) brain and spinal cord d) the somatic nerves

c

What is a benefit of a nerve plexus? a) The dorsal ramus can be distributed to many areas. b) A plexus does not allow for redistribution. c) Damage to one single branch of a plexus does not necessarily disrupt all motor information sent to a region. d) They provide a straight path from the spinal cord to target muscles.

c

f a motor neuron in the body were stimulated by an electrode placed about midpoint along the length of the axon ________. a) the impulse would move to the axon terminal only b) muscle contraction would occur c) the impulse would spread bidirectionally d) the impulse would move to the axon terminal only, and the muscle contraction would occur

c

During an absolute refractory period, the membrane

cannot respond to further stimulation

Which part of the brain processes inputs received from the cerebral motor cortex, brain stem nuclei, and various sensory receptors, and then uses this information to coordinate balance, posture, and somatic movement problems?

cerebellum

White matter (myelinated fibers) is found in all of the following locations except the ________.

cerebral cortex

The phrenic nerve is a branch from the __________.

cervical plexus

somatic sensory fibers

conducts impulses from skin, skeletal muscles, and joints to CNS

visceral sensory fibers

conducts impulses from visceral organs to CNS

Many neurons influence a few neurons

converging

A bit of dust blows into and touches the cornea of the eye. Which of the following is likely to happen? a) Stimulation of the facial nerve (VII) will be perceived as pain. b) Nothing, because there is no sensory information sent from the cornea. c) Stimulation of the optic nerve (II) will cause tears to flow from the lacrimal gland. d) Stimulation of the ophthalmic division of the trigeminal nerve (V) will cause blinking.

d

A second nerve impulse cannot be generated until ________. a) the Na ions have been pumped back into the cell b) proteins have been resynthesized c) all sodium gates are closed d) the membrane potential has been reestablished

d

Dermatome maps are useful to clinicians because ________. a) they show the routes of motor nerves b) they show doctors how to avoid striking spinal nerves during surgery c) they outline the location of the numerous nerve plexus d) they can help pinpoint the location of spinal injury

d

If the ventral root of a spinal nerve were cut, what would be the result in the tissue or region that nerve supplies? a) a complete loss of sensation and movement b) loss of neither sensation nor movement but only of autonomic control c) complete loss of sensation d) a complete loss of voluntary movement

d

If the permeability of the plasma membrane to K+ increases, resting membrane potential ____. This is called

decreases, hyperpolarization

The electrical change that occurs on a neuron membrane that receives a stimulus is called

depolarization

An action potential is self-regenerating because __________.

depolarizing currents established by the influx of Na+ flow down the axon and trigger an action potential at the next segment

One or a few inputs ultimately influence large numbers of neurons

diverging

one or a few inputs ultimately influence large numbers of neurons

diverging

peripheral nervous system

division of the nervous system that consists mainly of nerves that extend from the brain and spinal cord

Arachnoid granulations are knoblike projections that protrude superiorly through the ________ mater to absorb cerebrospinal fluid into venous blood.

dura

4) The synapse more common in embryonic nervous tissue than in adults is the ________.

electrical synapse

nervous system

employs electrical and chemical means to send messages from cell to cell

Which of the following neurotransmitters inhibits pain and is mimicked by morphine, heroin, and methadone?

endorphin

the neuropeptides that act as natural opiates are

enkephalines

The neuropeptides that act as natural opiates are

enkephalins

Which membrane potential occurs because of the influx of Na+ through chemically gated channels in the receptive region of a neuron?

excitatory postsynaptic potential

In a crossed-extensor reflex, if the right arm was grabbed it would flex and the left arm would ________.

extend

A patient who received a blow to the side of the skull exhibits the following signs and symptoms on that side of the face: he is unable to close his eye, and the corner of his mouth droops. Which cranial nerve has been damaged?

facial

Bipolar neurons are commonly

found in the retina of the eye

which ventricle is continuous with the central canal of the spinal cord?

fourth

What does the central nervous system use to determine the strength of a stimulus?

frequency of action potentials

At age 79, Mrs. X is diagnosed with a disorder that severely impairs her logical judgment. Medical imaging techniques show that this has been most likely caused by brain damage in a ______.

frontal lobe

sympathetic division

functional division of autonomic nervous system that mobilizes body systems during activity

Collections of nerve cell bodies outside the central nervous system are called

ganglia

Groups of neuron cell bodies in the peripheral nervous system are called

ganglia

When a sensory neuron is excited by some form of energy, the resulting graded potential is called a(n)

generator potential

Bundles of unmyelinated axons, dendrites, and neuron cell bodies in the CNS form

gray matter

Ridges of tissue on the surface of the cerebral hemispheres are called ________.

gyri

The velocity of nerve impulse conduction is greatest in

heavily mylinated, large-diameter fibers

An inhibitory postsynaptic potential (IPSP) is associated with

hyperpolarization

Decreasing the extracellular concentration of K+ affects the resting membrane potential by causing

hyperpolarization

An axon that is more negative than the resting membrane potential is said to be

hyperpolarized

A mugger steals your wallet causing all of the following to happen EXCEPT ________.

increased glucose uptake to the liver from blood

sensory input

information gathered by sensory receptors about internal and external changes

Which of the following is NOT required for a reflex arc?

interneurons

Chemical synapses are characterized by all of the following except

ions flowing through protein channels from the presynaptic to postsynaptic neuron What it is release of neurotransmitter by the presynaptic membranes, postsynaptic membranes bearing receptors that bind neurotransmitters, and a fluid-filled gap separating the neurons

Chemical synapses are characterized by all of the following except

ions flowing through protein channels from the presynaptic to the postsynaptic neuron

Parasympathetic functions include ________.

lens accommodation for close vision

In the presence of lidocaine, the action potential was NOT affected at R1 because

lidocaine was applied downstream of R1

Injury to the hypothalamus may result in all of the following EXCEPT ________.

loss of fine motor control

The typical concentration of sodium is

lower than potassium intracellularly

n carpal tunnel syndrome, the __________ is compressed.

median nerve

Vital centers for the control of heart rate, respiration, and blood pressure are located in the ________.

medulla oblongata

Meningitis can be caused by infection of the central nervous system by bacteria. Which cells would be most responsible for removing the infection?

microglia

The brain stem consists of the

midbrain, medulla, and pons

The brain stem consists of the _____________.

midbrain, medulla, and pons

Concerning concentration difference across the plasma membrane, there are

more K+ inside the cell than outside and more Na + outside the cell than inside

A neuron with many short dendrites and a single long axon is a ______ neuron.

multipolar

What structural classification describes this neuron?

multipolar

The white matter of the spinal cord contains ________.

myelinated and unmyelinated nerve fibers

Saltatory conduction refers to the conduction of impulses in

myelinated fibers

3) A chemical messenger that does not directly cause ESPSs or IPSPs but does affect the strength of synaptic transmission is a ________.

neuromodulator

Like all cells, the neurons' internal organization dictates its function. Neurons have relatively many mitochondria, an extensive network of rough endoplasmic reticulum and many clusters of ribosomes. These cellular features indicate all of the following EXCEPT

neurons have stable, relatively unchanging internal environments

Which receptor paring below is correct?

nicotinic; acetylchline

Increasing the voltage resulted in which of the following?

no change to the action potential

Biogenic amine neurotransmitters include

norepinephrine, dopamine, serotonin

Myelinates nerve fibers in the CNS

oligodendrocyte

What effect did decreasing the extracellular sodium have on the resting membrane potential?

only a small change occurred, because the resting neuron is not permeable to sodium

Which result of the stimulus applied is the likely cause of the response observed in the left graph?

opening of gated Na+ channels

Which of the following is expected to occur first if the membrane potential increase shown in the graph were to reach the threshold value indicated at -55 mV?

opening of voltage gated Na+ channels

5) When information is delivered within the CNS simultaneously by different parts of the neural pathway, the process is called ________ processing.

parallel

May be involved in exacting types of mental activity

parallel after-discharge

Which of the following is mismatched?

parasympathetic postganglionic fibers; release NE

Which of the meninges is a delicate connective tissue membrane that clings tightly to the brain like cellophane wrap following its every convolution?

pia mater

What regions of the brain stem serves as a bridge between the brain stem and the cerebellum?

pons

Compared to the inside of the resting plasma membrane, the outside surface of the membrane is

positively charged

Which motor area both has a homunculus and has descending projection fibers?

primary motor cortex

In order for an action potential to occur the cell membrane

reach threshold

The somatic and autonomic nervous systems differ in all of the following EXCEPT ________

regulation of activity by higher brain centers

Which of these activities is most likely driven by parasympathetic innervation?

resting and digesting

The box labeled D illustrates three mechanisms by which the effects of a neurotransmitter may be terminated. Which of the following mechanisms is NOT included in the figure?

reuptake of the neurotransmitter by transport into the postsynaptic cell

Impulses continue around and around the circuit until one neuron stops firing

reverberating

impulse continue around and around the circuit until one neuron stops firing?

reverberating

myelinates nerve fibers in the PNS

schwann cell

Afferent neurons can usually be classified as

sensory

Neurons that carry impulses from receptors to the CNS are called

sensory

Cranial nerve II, the optic nerve sends nerve impulses to the brain carrying information about the things we see. These nerve fibers most likely belong to which division of the nervous system?

sensory (afferent) division

dendrites

short, tapering, diffusely branched processes; receptive (input) region of neuron; convey incoming messages toward cell body as graded potentials (short distance signals)

The effects of lidocaine and tetrodotoxin were

similar, but tetrodotoxin had a greater effect

Which of these effectors is NOT directly controlled by the autonomic nervous system?

skeletal muscle

Hyperpolarization results from __________

slow closing of voltage-gated K+ channels

Establishing the resting membrane potential requires energy through the use of the

sodium potassium pump

6) When one or more presynaptic neurons fire in rapid order it produces a much greater depolarization of the postsynaptic membrane than would result from a single EPSP; this event is called ________ summation.

temporal

The primary auditory cortex is located in the ________.

temporal lobe

Which of the following blocks voltage-gated sodium channels?

tetrodotoxin and lidocaine

central nervous system

the brain and spinal cord of the dorsal body cavity are apart of what division of the nervous system

As a cook chops red onions he begins to tear up due to activation of the lacrimal gland. Which of the following nerves provided the stimulus?

the facial nerve

If a motor neuron in the body were stimulated by an electrode placed about midpoint along the length of the axon

the impulse would spread bidirectionally

A patient has lost vision on the left side of both eyes. The patient has likely suffered damage to ________.

the right optic tract

Synaptic Cleft

the small space between the sending neuron and the receiving neuron.

During the relative refractory period

the stimulus must be above threshold to generate action potentia

somatic and autonomic nervous systems

the two divisions of the motor division of the PNS

A doctor asks her patient to follow the motion of her finger as she moves it up and down, left and right. Which of the following cranial nerves is not being tested?

the vestibulocochlear nerve

During the relative refractory period of an action potential, a larger than normal stimulus is needed to cause another action potential. This is due to the fact that

the voltage gated potassium ion channels remain open long enough to hyperpolarize the axon membrane

motor division

transmits impulses from CNS to effector organs (muscles and glands)

The region on the neuron where action potentials are generated is called the

trigger zone

A postsynaptic potential is a graded potential that is the result of a neurotransmitter released into the synapse between two neurons.

true

Acetylcholine is released by all somatic motor neurons, all preganglionic neurons of the ANS and by the postganglionic parasympathetic nerve fibers. t or f?

true

Which of these materials or structures would be found in greatest amounts or numbers at E

vesicles containing neurotransmitter Neurotransmitters are released by secretion from the ends of axonal terminals

Problems in balance may follow trauma to which nerve?

vestibulocochlear

autonomic nervous system

visceral motor nerve fibers; regulates smooth muscle, cardiac muscle, and glands; involuntary nervous system

An individual who could trace a picture of a bicycle with his or her finger but could not recognize it as a bicycle is most likely to have sustained damage to the ________.

visual association area

Immediately after an action potential has peaked, which of the following channels will open?

voltage gated potassium channels

The depolarization phase of an action potential results from the opening of which channels?

voltage-gated Na+ channels

Which ion channel opens in response to a change in membrane potential and participates in the generation and conduction of action potentials?

voltage-gated channel

In which area of the neuron is an action potential initially generated?

C

Schwann cells are functionally similar to

oligodendrocytes

A) C B) B C) A 1. Which neuron would activate to a muscle? 2. Which neuron would be found in the retina of the eye? 3. Which neuron is a sensory neuron found in a reflex arc? 4. Which neuron is never myelinated? 5. Which neuron is typically involved in the special senses of sight and smell? 6. In a reflex arc, which neuron has its cell body inside the spinal cord? 7. Which neuron is common only in dorsal root ganglia of the spinal cord and sensory ganglia of cranial nerves? 8. Which is by far the most common neuron type?

1) C 2) B 3) A 4) B 5) B 6) C 7) A 8) C

A) C B) A C) B D) D 1) Myelin sheath. 2) Cell body of an ANS preganglionic neuron. 3) Cell body of an ANS postganglionic neuron. 4) Cell body of a somatic motor neuron.

1) C 2) B 3) A 4) D

(1.) Explain the difference between an EPSP and an IPSP. (2.) What specifically determines whether an EPSP and an IPSP will be generated at the postsynaptic membrane?

1. An EPSP is an excitatory (depolarizing) postsynaptic potential that increases the chance of a depolarization event. An IPSP is an inhibitory hyperpolarizing) postsynaptic potential that decreases the chance of a depolarization event. 2. EPSPs and IPSPs are determined by the type and amount of neurotransmitter that binds at the postsynaptic neuron and the specific receptor subtype it binds to.

The PNS contains a. 12 pairs of cranial nerves only b. 31 pairs of spinal nerves only c. 12 pairs of cranial nerves and 31 pairs of spinal nerves d. 43 pairs of spinal nerves

12 pairs of cranial nerves and 31 pairs of spinal nerves

A) Depolarization B) Action potential C) Relative refractory period D) Absolute refractory period E) Repolarization 14. The neuron cannot respond to a second stimulus, no matter how strong. 15. The interior of the cell becomes less negative due to an influx of sodium ions. 16. The specific period during which potassium ions diffuse out of the neuron due to a change in membrane permeability. 17. Also called a nerve impulse transmitted by axons. 18. An exceptionally strong stimulus can trigger a response.

14) D 15) A 16) E 17) B 18) C

A) C B) B C) A D) D E) E 16) Auricle. 17) Tympanic membrane. 18) Stapes. 19) Semicircular canals. 20) Cochlea. 21) Balance organ. 22) Houses organ of Corti.

16) C 17) E 18) A 19) B 20) D 21) B 22) D

A) Plantar B) Flexor C) Crossed-extensor D) Tendon E) Stretch 17) Tests both upper and lower motor pathways. The sole of the foot is stimulated with a dull instrument. 18) Consists of an ipsilateral withdrawal reflex and a contralateral extensor reflex; important in maintaining balance. 19) Produces a rapid withdrawal of the body part from a painful stimulus; ipsilateral. 20) Prevents muscle overstretching and maintains muscle tone. 21) Produces muscle relaxation and lengthening in response to tension; the contracting muscle relaxes as its antagonist is activated.

17) A 18) C 19) B 20) E 21) D

A) Insula B) Parietal lobe C) Frontal lobe D) Temporal lobe E) Occipital lobe 18) Auditory area. 19) Primary somatosensory cortex. 20) Primary (somatic) motor cortex. 21) Motor speech (Broca's) area. 22) Premotor cortex. 23) Visual area. 24) Gustatory (taste) area. 25) Seat of intelligence, abstract reasoning.

18) D 19) B 20) C 21) C 22) C 23) E 24) A 25) C

A) Spatial summation B) Subthreshold stimulus C) Threshold stimulus D) Temporal summation 19. Numerous nerve impulses arriving at a synapse at closely timed intervals exert a cumulative effect 20. Stimulation of a postsynaptic neuron by many terminals at the same time. 21. An insufficient stimulus to reach an action potential. 22. Any stimulus at or above this level will result in action potential.

19) D 20) A 21) B 22) C

A) Lens B) Retina C) Aqueous humor D) Fovea centralis 23) The sensory layer of the eye. 24) The structure most responsible for focusing light rays that enter the eye. 25) Helps maintain the intraocular pressure; located in the anterior part of the eye. 26) Area of greatest visual acuity.

23) B 24) A 25) C 26) D

A) Secretory zone B) Conducting region C) Secretory region D) Trigger zone E) Receptive region 23. Area where nerve impulse is generated. 24. Receives stimuli. 25. Plasma membrane exhibits voltage-gated Na+ and K+ channels. 26. Plasma membrane exhibits voltage-gated Ca2+ channels. 27. Axon terminals release neurotransmitters. 28. Plasma membrane exhibits chemically gated ion channels.

23) D 24) E 25) B 26) C 27) A 28) E

A) Otoliths B) Vestibule C) Pharyngotympanic tube D) Tympanic membrane 27) Ear stones. 28) Connects the middle ear with the nasopharynx. 29) Separates external acoustic meatus from the middle ear. 30) Contains utricle and saccule. 31) Detects linear acceleration.

27) A 28) C 29) D 30) B 31) B

A) Parallel after-discharge circuit B) Reverberating circuit C) Diverging circuit D) Converging circuit 29. One incoming axon triggers responses in ever-increasing numbers farther and farther along the circuit. 30. May be involved in complex, exacting types of mental processing 31. Involved in control of rhythmic activities such as breathing. 32. Involved in activating fibers of a skeletal muscle such as the biceps muscle. 33. Different types of sensory input can have the same ultimate effect.

29) C 30) A 31) B 32) C 33) D

A) NREM stage 2 B) REM C) NREM stage 1 D) NREM stage 4 E) NREM stage 3 30) The stage when vital signs (blood pressure, heart rate, and body temperature) reach their lowest normal levels. 31) Indicated by movement of the eyes under the lids; dreaming occurs. 32) Theta and delta waves begin to appear. 33) Very easy to awaken; EEG shows alpha waves; may even deny being asleep. 34) Typified by sleep spindles. 35) Begins about 90 minutes after the onset of sleep. 36) May allow the brain to work through emotional problems in dream imagery.

30) D 31) B 32) E 33) C 34) A 35) B 36) B

A) Conduction deafness B) Otitis media C) Night blindness D) Glaucoma 32) A condition that can result from a deficiency of vitamin A. 33) A condition of deafness that may result from otosclerosis. 34) An inflammation of the lining of the middle ear. 35) A condition often leading to blindness due to increased intraocular pressure.

32) C 33) A 34) B 35) D

A) Macula B) Taste buds C) Olfactory epithelium D) Crista ampullaris 36) Hair cells receptive to changes in dynamic equilibrium are found in this structure. 37) Chemically sensitive microvilli found in this structure. 38) Hair cells receptive to changes in static equilibrium are found in this structure. 39) Chemically sensitive cilia are found in this structure.

36) D 37) B 38) A 39) C

A) Brain stem B) Cerebellum C) Hypothalamus D) Cerebrum E) Thalamus 37) Gateway to the cerebrum. 38) Ensures coordination and balance. 39) Produces automatic behaviors necessary for survival. 40) Executive suite. 41) Visceral command center.

37) E 38) B 39) A 40) D 41) C

A) Pacinian corpuscle B) Ruffini endings C) Meissner corpuscle 38) Lamellar corpuscle. 39) Bulbous corpuscle. 40) Tactile corpuscle.

38) A 39) B 40) C

A) Spiral organ (of Corti) B) Cochlear duct C) Vestibule D) Tympanic membrane 40) Houses the spiral organ (of Corti). 41) The receptor organ for hearing. 42) The central part of the bony labyrinth. 43) A membrane that transmits sound vibrations to the auditory ossicles.

40) B 41) A 42) C 43) D

A) Conus medullaris B) Cauda equina C) Filum terminale D) Cervical enlargement 42) Where nerves serving the upper limbs arise. 43) Anchors the spinal cord to the coccyx. 44) Collection of nerve roots at the inferior end of the vertebral canal. 45) Inferior point of termination of the spinal cord in an adult.

42) D 43) C 44) B 45) A

A) Tinnitus B) Sensorineural deafness C) Conduction deafness 44) Loss of hearing resulting from prolonged exposure to high-intensity sounds. 45) Can result from the fusion of the auditory ossicles. 46) A possible side effect of medications such as aspirin. 47) One of the most common results of otitis media. 48) Can result from impacted cerumen.

44) B 45) C 46) A 47) C 48) C

A) D B) C C) E D) A E) B 6) Protects and shapes the eyeball; provides a sturdy anchoring site for extrinsic eye muscles. 7) Blood vessels that supply nutrition to most of the eyes' structures (excluding the lens and cornea). 8) Contains only cones; provides detailed color vision. 9) Lacks photoreceptors; where optic nerve exits the eye. 10) Consists of a pigmented layer and a neural layer.

6) D 7) E 8) A 9) C 10) B

A) A B) B C) C D) D E) E 6) Pons. 7) Corpus callosum. 8) Caudate nucleus. 9) Globus pallidus. 10) Thalamus.

6) E 7) A 8) B 9) C 10) D

A) E B) C C) A D) B E) D 7) Integration center. 8) Sensory neuron. 9) Effector. 10) Motor neuron. 11) Receptor.

7) A 8) B 9) D 10) E 11) C

A) C B) E C) B D) D E) A 9. Ion channel. 10. Synaptic vesicles. 11. Calcium ions. 12. Postsynaptic membrane. 13. Synaptic cleft.

9) B 10) A 11) E 12) C 13) D

A patient is admitted to the hospital with exacerbation of multiple sclerosis (MS). She asks the nurse "Why did this have to happen to me again? I was doing so well." Why are some forms of MS characterized by periods of remission and exacerbation? A)The axons are not damaged; growing numbers of sodium channels appear spontaneously in the demyelinated fibers, allowing conduction to resume. B) The myelin sheaths are damaged, and repair of the myelin sheaths results in impulse propagation resuming. C) The axons are damaged, and repair of the axons results in impulse propagation resuming. D) The axons are damaged, and repair of the myelin sheath results in impulse propagation resuming. E) The axons are not damaged; growing numbers of potassium channels appear spontaneously in the demyelinated fibers, allowing conduction to resume.

A

3) Which of the following is the best explanation of why it is difficult to discriminate the color of an object at night? A) Rods contain a single kind of visual pigment. B) Cones come in three types, each sensitive to different wavelengths of light. C) The foveae are densely packed with cones. D) As many as 100 cones may converge on one ganglion cell.

A) Rods contain a single kind of visual pigment.

How would an increase in extracellular K+ affect repolarization? A) It will decrease the concentration gradient, causing less K+ to flow out of the cell during repolarization. B) It will not have any effect; only ions inside the cell change the cell's charge. C) It will increase the concentration gradient, causing less K+ to flow out of the cell during repolarization. D) It will decrease the concentration gradient, causing more K+ to flow out of the cell during repolarization.

A

Which of the following types of glial cells monitors the health of neurons, and can transform into a special type of macrophage to protect endangered neurons? A) microglia B) oligodendrocytes C) astrocytes D) ependymal cells

A

B) neurotransmitter.

A molecule that carries information across a synaptic cleft is a A) synaptic cleft. B) neurotransmitter. C) receiving neuron. D) sending neuron. E) synapse.

What is the polarized membrane state? How is it maintained?

A polarized membrane possesses a net positive charge outside, and a net negative charge inside, with the voltage across the membrane being at -70 mV. Diffusion of Na+ and K+ across the membrane establishes the resting potential because the membrane is slightly more permeable to K+. The Na+-K+ pump, an active transport mechanism, maintains this polarized state by maintaining the diffusion gradient for Na+ and K+.

If a post synaptic cell is stimulated to threshold by temporal summation this implies that

A presynaptic neuron is sending frequent EPSP

An action potential is regarded as an example of a positive feedback. Which of the following examples below best illustrates the positive feedback aspect of an action potential?

A threshold stimulus will cause the opening of voltage gated sodium ion channels that will cause further depolarizing stimulus. This stimulus will open still more voltage gated sodium ion channels.

12) Color vision has much greater resolution than night vision (vision that is mostly in shades of gray). Which of the following is the best explanation for why this is so? A) A single cone photoreceptor often connects to a single bipolar cell and a single ganglion cell while as many as 100 rods will converge to a single ganglion cell. B) There are many more cone photoreceptors in the eye than rod photoreceptors. C) There are several types of cone photoreceptors, each of which enrich the clarity and resolution of vision. D) A larger proportion of the brain's visual cortex is active during the day when our cone photo receptors are most active.

A) A single cone photoreceptor often connects to a single bipolar cell and a single ganglion cell while as many as 100 rods will converge to a single ganglion cell.

11) Humans can see several thousand shades of color but have cone photoreceptors that are sensitive to only three (perhaps four) wavelengths of light. What is the best explanation for why we see so many colors? A) Color perception is achieved by activation of various combinations between the three cone types. B) Color perception is dependent on the millions of rods as well as cone photoreceptors. C) Shades of color are purely psychological and learned by association with age, infants only seeing in black and white. D) Colors are added and enhanced in the primary visual cortex of the brain.

A) Color perception is achieved by activation of various combinations between the three cone types.

38) Which statement about coma is true? A) Coma may be caused by widespread cerebral or brain stem trauma. B) During coma, brain oxygen consumption resembles that of a waking state. C) Coma is a form of deep sleep. D) Coma is neurologically identical to syncope.

A) Coma may be caused by widespread cerebral or brain stem trauma.

33) Which of the following is true about gustatory receptors? A) Complete adaptation occurs in about one to five minutes. B) All gustatory receptors have the same threshold for activation. C) The receptors generate an action potential in response to chemical stimuli. D) In order for a chemical to be sensed, it must be hydrophobic.

A) Complete adaptation occurs in about one to five minutes.

43) Select the statement that is most correct. A) Ganglia associated with afferent nerve fibers contain cell bodies of sensory neurons. B) The dorsal root ganglion is a motor-only structure. C) The cell bodies of afferent ganglia are located in the spinal cord. D) Ganglia are collections of neuron cell bodies in the spinal cord that are associated with efferent fibers.

A) Ganglia associated with afferent nerve fibers contain cell bodies of sensory neurons.

17) Bill is a mechanic that works with vibrating tools. He also exerts force on his wrists when twisting wrenches and screws. Bill has a tingling sensation in the lateral portion of his hand. The doctor suspects carpal tunnel syndrome. Which of the following test might the doctor try on her patient? A) Have bill grip an object with his thumb and index finger and try to pull the object away. B) Check for hyperextension at the knuckles of the little and ring finger. C) Have bill flex and extend his arm against resistance. D) Check for reflex on the medial condyle of the humerus with a rubber mallet.

A) Have bill grip an object with his thumb and index finger and try to pull the object away.

A) Contrast uni-polar, bipolar, and multi-polar neurons structurally. B) Indicate where each is most likely to be found.

A) Multi-polar have 3 or more processes. B) Multi-polar neurons are the most common type in humans. Major neuron in CNS. A) Bipolar have 2 processes, an axon and a dendrite. B) Bipolar neurons are rare neurons found in some of the special sense organs. Example: neurons in the retina of the eye and olfactory mucosa. A) Uni-polar neurons have a single short process. More accurately called pseudounipolar neurons because they originate as bipolar neurons. The 2 processes converge and partially fuse to form one single process during embryonic development. B) Uni-polar neurons are found in the ganglia in the PNS, where they function as sensory neurons.

60) Sleepwalking may occur during ________. A) NREM stage 4 B) NREM stage 1 C) NREM stage 2 D) REM

A) NREM stage 4

39) Tremor at rest, shuffling gait, stooped posture, and expressionless face are characteristics of ________. A) Parkinson's disease B) Alzheimer's disease C) cerebellar disease D) Huntington's disease

A) Parkinson's disease

13) Which of the following generalizations does NOT describe the cerebral cortex? A) The hemispheres are exactly equal in function. B) No functional area of the cortex works alone. C) Each hemisphere is chiefly concerned with sensory and motor functions of the contralateral side of the body. D) The cerebral cortex contains three kinds of functional areas.

A) The hemispheres are exactly equal in function.

17) A patient has a loss of hearing in only one ear. Which of the following is likely to be a result? A) The patient will not be able to localize the origin of sounds. B) The patient will have increased sensitivity to sound in the unaffected ear. C) The patient will have a loss of balance with dizziness and vertigo. D) The patient will not be able to track objects with the eye on the same side as the hearing loss.

A) The patient will not be able to localize the origin of sounds.

33) Select the correct statement regarding chemical synapses. A) The synaptic cleft uses the action potential to transmit a chemical signal to the post synaptic cell. B) The release of neurotransmitter molecules gives cells the property of being electrically coupled. C) Cells with gap junctions use chemical synapses. D) Neurotransmitter receptors are located on the axons terminals of cells.

A) The synaptic cleft uses the action potential to transmit a chemical signal to the post synaptic cell.

40) Which of the following is not true of graded potentials? A) They increase amplitude as they move away from the stimulus point. B) They can be called postsynaptic potentials. C) They can form on receptor endings. D) They are short-lived.

A) They increase amplitude as they move away from the stimulus point.

11) The depolarization phase of an action potential is punctuated by the closing of inactivation gates in the voltage gated sodium ion channels. All of the following are consequences of this inactivation except one. Choose the statement below that is not a consequence of the closing of inactivating gates. A) This allows for the efflux (diffusion out) of potassium ions, resulting in the repolarization of the cell. B) This allows for the one way transmission of action potential down the axon. C) This limits the frequency of action potentials down the axon. D) This stops the depolarization of the axon membrane.

A) This allows for the efflux (diffusion out) of potassium ions, resulting in the repolarization of the cell.

10) Select the statement below that is not true with regards to the process of light adaptation. A) Visual acuity is diminished. B) Retinal sensitivity decreases. C) Rhodopsin is uncoupled from light transduction. D) The activity of rods is reduced by rapid bleaching of rhodopsin.

A) Visual acuity is diminished.

29) The term central nervous system refers to the ________. A) brain and spinal cord B) sensory (afferent) nerves C) the somatic nerves D) the spinal nerves

A) brain and spinal cord

44) Information from balance receptors goes directly to the ________. A) brain stem reflex centers B) visual cortex C) motor cortex D) back muscles

A) brain stem reflex centers

44) White matter is found in all of the following locations EXCEPT the ________. A) cerebral cortex B) outer portion of the spinal cord C) corpus callosum D) corticospinal tracts

A) cerebral cortex

37) Bell's palsy is ________. A) characterized by paralysis of facial muscles B) often caused by inflammation of the trigeminal nerve C) characterized by loss of vision D) characterized by partial paralysis of diaphragm muscles

A) characterized by paralysis of facial muscles

62) Damage to the medial rectus muscles would probably affect ________. A) convergence B) pupil constriction C) accommodation D) refraction

A) convergence

19) Tom is a 45-year-old male that has lost his ability to hear high frequency sounds. The most likely explanation for this would be ________. A) damage to the hair cells near the oval window in the cochlear duct B) middle ear infection C) an overgrowth of bony tissue, fusing the ossicles together D) a perforated tympanic membrane

A) damage to the hair cells near the oval window in the cochlear duct

23) Drugs called beta-blockers ________. A) decrease heart rate and blood pressure B) have widespread sympathetic effects C) increase a dangerously low heart rate D) are potent antidepressants

A) decrease heart rate and blood pressure

58) Which type of wave predominates during stage 4 NREM sleep? A) delta B) alpha C) beta D) theta

A) delta

28) Collections of nerve cell bodies outside the central nervous system are called ________. A) ganglia B) nerves C) tracts D) nuclei

A) ganglia

26) Loss of function in the enzyme acetylcholine esterase would result in which of the following? A) inability to destroy and remove acetylcholine from the synaptic cleft B) stimulation of the production of acetylcholine C) amplify or enhance the effect of ACh D) inability to release acetylcholine

A) inability to destroy and remove acetylcholine from the synaptic cleft

31) Sympathetic division stimulation causes ________. A) increased blood glucose, decreased GI peristalsis, and increased heart rate and blood pressure B) decreased blood glucose, increased GI peristalsis, and increased heart rate and blood pressure C) increased blood glucose, increased GI peristalsis, and decreased heart rate and blood pressure D) decreased blood glucose, increased GI peristalsis, and decreased heart rate and blood pressure

A) increased blood glucose, decreased GI peristalsis, and increased heart rate and blood pressure

59) The eye muscle that elevates and turns the eye laterally is the ________. A) inferior oblique B) medial rectus C) superior oblique D) lateral rectus

A) inferior oblique

16) Complicated interlacing of the ventral rami form networks called nerve plexus. The crisscrossing of the nerve fibers from the various spinal nerves is advantageous because ________. A) injury to any single spinal nerve will be less damaging as there is less chance of total loss of innervation to any particular organ B) having several nerve fibers unite enhances sensory function C) viruses that infect us by moving through PNS nerves are prevented entry to the CNS D) having several nerve fibers unite enhances motor function

A) injury to any single spinal nerve will be less damaging as there is less chance of total loss of innervation to any particular organ

25) Inborn or intrinsic reflexes are ________. A) involuntary, yet may be modified by learned behavior B) always mediated by the brain C) autonomic only D) rapid, predictable, and can be learned responses

A) involuntary, yet may be modified by learned behavior

40) Dark adaptation ________. A) involves accumulation of rhodopsin B) results in inhibition of rod function C) is much faster than light adaptation D) primarily involves improvement of acuity and color vision

A) involves accumulation of rhodopsin

31) Regeneration within the CNS ________. A) is prevented due to growth-inhibiting proteins of oligodendrocytes B) is more successful than with the PNS C) is promoted by growth inhibitors and glial scars D) typically allows axonal sprouting of 20 mm

A) is prevented due to growth-inhibiting proteins of oligodendrocytes

41) Declarative memory ________. A) is the ability to learn specific information B) usually involves motor skills C) is best remembered in the doing D) is hard to unlearn when learned once

A) is the ability to learn specific information

34) Injury to the hypothalamus may result in all of the following EXCEPT ________. A) loss of fine motor control B) loss of body temperature control C) sleep disturbances D) dehydration

A) loss of fine motor control

23) The receptor for static equilibrium is the ________. A) macula B) cochlear duct C) utricle D) semicircular canals

A) macula

18) Which of the choices below describes the ANS? A) motor fibers that conduct nerve impulses from the CNS to smooth muscle, cardiac muscle, and glands B) motor fibers that conduct nerve impulses from the CNS to skeletal muscles C) sensory and motor neurons that supply the digestive tract D) sensory neurons that convey information from somatic receptors in the head, body wall, and limbs and from receptors from the special senses of vision, hearing, taste, and smell to the CNS

A) motor fibers that conduct nerve impulses from the CNS to smooth muscle, cardiac muscle, and glands

18) The white matter of the spinal cord contains ________. A) myelinated and unmyelinated nerve fibers B) unmyelinated nerve fibers only C) cell bodies D) myelinated nerve fibers only

A) myelinated and unmyelinated nerve fibers

21) Which of the follow types of neurons are replaced throughout adult life? A) olfactory receptor cells B) auditory outer and inner hair cells C) retinal ganglion cells D) retinal bipolar cells

A) olfactory receptor cells

49) The first vestiges of eyes in the embryo are called ________. A) optic vesicles B) mesenchyme C) optic discs D) optic cups

A) optic vesicles

8) The "resting and digesting" division of the autonomic nervous system is the ________. A) parasympathetic division B) somatic division C) peripheral division D) sympathetic division

A) parasympathetic division

46) Loss of ability to perform skilled motor activities such as piano playing, with no paralysis or weakness in specific muscles, might suggest damage to the ________. A) premotor cortex B) primary motor cortex C) rubrospinal tracts D) spinal cord

A) premotor cortex

42) In the visual pathways to the brain, the optic radiations project to the ________. A) primary visual cortex B) medial retina C) lateral geniculate body D) optic chiasma

A) primary visual cortex

38) Which of the following is mismatched? A) promotes urination; sympathetic B) gallbladder contracts to expel bile; parasympathetic C) bronchiole dilation in lungs; sympathetic D) penis ejaculation; sympathetic

A) promotes urination; sympathetic

38) Which of the following is the correct simple spinal reflex arc? A) receptor, afferent neuron, integration center, efferent neuron, effector B) receptor, efferent neuron, integration center, afferent neuron, effector C) effector, afferent neuron, integration center, efferent neuron, receptor D) effector, efferent neuron, integration center, afferent neuron, receptor

A) receptor, afferent neuron, integration center, efferent neuron, effector

22) Which of the following appears to exert the most direct influence over autonomic functions? A) reticular formation B) hypothalamus C) midbrain D) medulla oblongata

A) reticular formation

30) The brain area that regulates activities that control the state of wakefulness or alertness of the cerebral cortex is the ________. A) reticular formation B) limbic system C) thalamus D) pyramids

A) reticular formation

44) A fall or an improperly delivered gluteal injection could result in ________. A) sciatica B) neurofibromatosis C) postpoliomyelitis muscular atrophy D) phantom limb pain

A) sciatica

4) Cranial nerve II, the optic nerve sends nerve impulses to the brain carrying information about the things we see. These nerve fibers most likely belong to which division of the nervous system? A) sensory (afferent) division B) somatic nervous system C) sympathetic division D) parasympathetic division

A) sensory (afferent) division

9) If you shine a light into one eye both pupils will constrict. The best explanation for this is ________. A) sensory input from the retinas of both eyes converges at the optic chiasm and information from each eye is delivered to both the left and right sides of the brain B) information from one eye is directly and immediately transferred to the other eye to maintain alignment of the eyes C) a small portion of light always enters the other eye D) this in fact does not occur and information from both eyes is always separated

A) sensory input from the retinas of both eyes converges at the optic chiasm and information from each eye is delivered to both the left and right sides of the brain

40) Which target organ receives dual innervation? A) small intestine B) kidney C) sweat glands D) coronary blood vessels

A) small intestine

43) Visual inputs to the ________ serve to synchronize biorhythms with natural light and dark. A) suprachiasmatic nucleus B) superior colliculi C) lateral geniculate body D) pretectal nuclei

A) suprachiasmatic nucleus

36) What prevents the eyelids from sticking together when the eyes close? A) tarsal gland secretions B) lacrimal fluid C) ciliary gland secretions D) conjunctival fluid

A) tarsal gland secretions

15) Autonomic ganglia contain ________. A) the cell bodies of motor neurons B) both somatic afferent and efferent neurons C) synapses between postganglionic fibers and their effectors D) an outer connective tissue capsule around the cell bodies of preganglionic motor neurons

A) the cell bodies of motor neurons

12) During the relative refractory period of an action potential, a larger than normal stimulus is needed to cause another action potential. This is due to the fact that ________. A) the voltage gated potassium ion channels remain open long enough to hyperpolarize the axon membrane B) the membrane is now impermeable to all ions C) the sodium potassium pump will stop working during relative refractory D) the inactivation gates on voltage gated sodium ion channels are closed

A) the voltage gated potassium ion channels remain open long enough to hyperpolarize the axon membrane

20) What is the main function of the rods in the eye? A) vision in dim light B) accommodation for near vision C) depth perception D) color vision

A) vision in dim light

Inhibition of acetylcholinesterase by poisoning blocks neurotransmission at the neuromuscular junction because:

ACh is not degraded, hence prolonged depolarization is enforced on the postsynaptic ACh receptors

Inhibition of acetylcholinesterase by poisoning blocks neurotransmission at the neuromuscular junction because

ACh is not degraded, hence prolonged depolarization is enforced on the postsynaptic cell

Inhibition of acetylcholinesterase by poisoning blocks neurotransmission at the neuromuscular junction because

ACh is not degraded, hence prolonged depolarization is enforced on the postsynaptic neuron

Inhibition of acetylcholinesterase by poisoning blocks neurotransmission at the neuromuscular junction because

ACh is not degraded, hence prolonged depolarized is enforced on the postsynaptic cell

Explain both the anatomical and functional divisions of the nervous system. Include the subdivisions of each.

Anatomical divisions are defined by their location in the body, and include the CNS (brain and spinal cord) and the PNS (nerves and ganglia outside the CNS). Functional divisions are defined by the type of information they relay between other structures in the body and the CNS. They include the sensory division of the PNS that relays signals from receptors to the CNS, and the somatic and autonomic motor divisions of the PNS that carry signals from the CNS to muscle or gland effector cells. The autonomic division is divided into sympathetic and parasympathetic subdivisions.

D. the sympathetic division of the autonomic nervous system

As you start working out, you notice that your heart rate and breathing rate start to increase. Which division of your nervous system is generating this response? Be as specific as possible. ANSWER: A. the somatic nervous system B. the afferent division of the nervous system C. the parasympathetic division of the autonomic nervous system D. the sympathetic division of the autonomic nervous system

Which glial cells have the most diversity of function?

Astrocytes are the most abundant and diversely functioning glial cells. Examples of their many functions include: anchoring neurons to capillaries, aiding in the exchanges between neurons and blood, guiding the migration of young neurons, and helping control the chemical environment around neurons.

What type of cells are found in the CNS?

Astrocytes, microglia, oligodendrocytes, ependyma

B Voltage-gated Na+ channels open when the membrane potential reaches threshold. Voltage-gated K+ channels would be mostly open near C.

At which point of the illustrated action potential would voltage-gated Na+ channels be mostly open but voltage-gated K+ channels be mostly closed? A B C D

Myelinated Axon

Axon with myelin sheath around it

An inhibitory postsynaptic potential (IPSP) is associated with ________. A) lowering the threshold for an action potential to occur B) hyperpolarization C) opening of voltage-regulated channels D) a change in sodium ion permeability

B

Are biogenic amine neurotransmitters such as dopamine considered to be proteins? A) No, dopamine cannot be a protein since proteins do not exhibit neurotransmitter activity. B) No, it simply means that dopamine has at least one amine functional group in its molecular structure. C) Yes, dopamine is a protein because of its expression of a nitrogen-containing amine group. D) Yes, amines are naturally occurring organic compounds composed of amino acids.

B

Cold sores on the skin of the mouth occur when herpes simplex viruses that are dormant in neural ganglia become active and travel to the skin of the mouth. Which of the following is the mechanism by which these viruses travel from the ganglia (located within the head) to the skin of the mouth? A) transport along nerve impulses that travel down the axons B) anterograde axonal transport C) retrograde axonal transport D) travel of the viruses along neurofibrils

B

The concentration neurotransmitters in the fluid of the synaptic cleft must be tightly regulated for neurons to function properly. Which of the following cells is most responsible for aiding in this regulation? A) satellite cells B) astrocytes C) Schwann cells D) oligodendrocytes

B

The generation of an action potential in a neuron requires the presence what type of membrane channels? A) leakage channels B) voltage-gated channels C) chemically gated channels D) membrane channels are not required

B

Which of the following circuit types is involved in the control of rhythmic activities such as the sleep-wake cycle, breathing, and certain motor activities (such as arm swinging when walking)? A) parallel after-discharge circuits B) reverberating circuits C) converging circuits D) diverging circuits

B

Which of the following will occur when an excitatory postsynaptic potential (EPSP) is being generated on the dendritic membrane? A) Sodium gates will open first, then close as potassium gates open. B) A single type of channel will open, permitting simultaneous flow of sodium and potassium. C) Specific potassium gates will open. D) Specific sodium gates will open.

B

At which point of the illustrated action potential would voltage-gated Na+ channels be mostly open but voltage-gated K+ channels be mostly closed?

B Voltage-gated Na+ channels open when the membrane potential reaches threshold. Voltage-gated K+ channels would be mostly open near C

38) Which of the following will occur when an excitatory postsynaptic potential (EPSP) is being generated on the dendritic membrane? A) Sodium gates will open first, then close as potassium gates open. B) A single type of channel will open, permitting simultaneous flow of sodium and potassium. C) Specific sodium gates will open. D) Specific potassium gates will open.

B) A single type of channel will open, permitting simultaneous flow of sodium and potassium.

8) Vital centers for the control of heart rate, respiration, and blood pressure are located in the ________. A) cerebrum B) medulla oblongata C) midbrain D) pons

B) medulla oblongata

57) The corpora quadrigemina are found in the ________. A) pons B) midbrain C) diencephalon D) cerebellum

B) midbrain

14) Flavor preferences, our likes and dislikes are said to have a "homeostatic value". Which of the following is the best example of this homeostatic value for taste preferences? A) The temperature, smell and texture of foods will influence its overall taste. B) Alkaloids are chemical compounds in plants that are frequently toxic "antifeedant" chemicals. Alkaloids are often unpleasantly bitter. C) Capsaicin is a chemical compound in chili peppers. Nociceptors in the mouth respond to this chemical with the sensation of heat. D) Monosodium glutamate is a food additive that stimulates umami taste (gustatory) cells.

B) Alkaloids are chemical compounds in plants that are frequently toxic "antifeedant" chemicals. Alkaloids are often unpleasantly bitter.

5) Which of the following is the best explanation for our perception of color? A) Rods contain a single kind of visual pigment. B) Cones come in three types, each sensitive to different wavelengths of light. C) The foveae are densely packed with cones. D) As many as 100 cones may converge on one ganglion cell.

B) Cones come in three types, each sensitive to different wavelengths of light.

33) Select the correct definition. A) Magnitude estimation is the simplest level of sensation. B) Pattern recognition allows us to see a familiar face. C) Perceptual detection is the ability to detect how much stimulus is applied to the body. D) Spatial discrimination allows us to recognize textures.

B) Pattern recognition allows us to see a familiar face.

41) Which of the following is true about the movement of ions across excitable living membranes? A) Ions always move actively across membranes through leakage channels. B) Sodium gates in the membrane can open in response to electrical potential changes. C) Ions always move from an area of higher concentration to an area of lower concentration. D) Ions always move passively across membranes.

B) Sodium gates in the membrane can open in response to electrical potential changes.

12) Which of the following statements is FALSE? A) Sympathetic has extensive branching of preganglionic fibers; parasympathetic has minimal branching of preganglionic fibers. B) Sympathetic origin is craniosacral; parasympathetic is thoracolumbar. C) Sympathetic division has short preganglionic and long postganglionic fibers; parasympathetic has long preganglionic and short postganglionic fibers. D) Sympathetic ganglia are within a few centimeters of the CNS; parasympathetic ganglia are close to the visceral organs served.

B) Sympathetic origin is craniosacral; parasympathetic is thoracolumbar.

22) Which of the following is an incorrect statement regarding the occurrence of a sensation? A) A generator potential in the associated sensory neuron must reach threshold. B) The stimulus energy must be converted into the energy of a graded potential called a transduction potential. C) The stimulus energy must match the specificity of the receptor. D) The stimulus energy must occur within the receptor's receptive field.

B) The stimulus energy must be converted into the energy of a graded potential called a transduction potential.

Which of the following is not a part of the central nervous system? A) the brain B) a nerve C) spinal cord D) a tract

B) a nerve

1) Which of the following does NOT describe the ANS? A) a system of motor neurons that innervates smooth and cardiac muscle and glands B) a system of motor neurons that innervates all muscle cells C) general visceral motor system D) involuntary nervous system

B) a system of motor neurons that innervates all muscle cells

5) The concentration of ions in the chemical environment surrounding the neurons must be tightly regulated for neurons to function properly. Which of the following cells is most responsible for this? A) satellite cells B) astrocytes C) Schwann cells D) oligodendrocytes

B) astrocytes

6) The concentration neurotransmitters in the fluid of the synaptic cleft must be tightly regulated for neurons to function properly. Which of the following cells is most responsible for aiding in this regulation? A) satellite cells B) astrocytes C) Schwann cells D) oligodendrocytes

B) astrocytes

40) Which of the following is the mildest consequence of traumatic brain injury? A) swelling B) concussion C) contusion D) hemorrhage

B) concussion

52) Arachnoid granulations are knoblike projections that protrude superiorly through the ________ mater to absorb cerebrospinal fluid into venous blood. A) subarachnoid B) dura C) arachnoid D) pia

B) dura

54) Which type of memory is exemplified by a racing heartbeat upon hearing a rattlesnake nearby? A) declarative (fact) B) emotional C) motor D) procedural (skills)

B) emotional

11) Which of the following best describes the cerebrum? A) motor command center B) executive suite C) visceral command center D) decussation center

B) executive suite

23) A major nerve of the lumbar plexus is the ________. A) ilioinguinal B) femoral C) sciatic D) iliohypogastric

B) femoral

50) Which pairing of terms is incorrectly related? A) amplitude of a sound: intensity of the sound B) frequency of sound waves: loudness of the sound C) frequency of sound waves: number of wavelengths D) quality of a sound : frequency of the sound

B) frequency of sound waves: loudness of the sound

30) There are three layers of neurons in the retina. The axons of which of these neuron layers form the optic nerves? A) rod cells B) ganglion cells C) bipolar cells D) cone cells

B) ganglion cells

34) Emotions influence autonomic reactions primarily through integration in the ________. A) lateral geniculate of the thalamus B) hypothalamus C) lateral horn of the spinal cord D) inferior colliculus

B) hypothalamus

34) Taste buds are not found ________. A) in circumvallate papillae B) in filiform papillae C) lining the buccal cavity D) in fungiform papillae

B) in filiform papillae

37) A mugger steals your wallet causing all of the following to happen EXCEPT ________. A) inability to read close-up print B) increased glucose uptake to the liver from blood C) increased rate and force of heartbeat D) increased metabolic rate

B) increased glucose uptake to the liver from blood

36) Which of the following structures is probably NOT directly involved in memory? A) prefrontal cortex B) medulla oblongata C) thalamus D) hippocampus

B) medulla oblongata

3) We can touch our finger to our nose while our eyes are closed in part because we can sense the position and movement of our joints as well as the length of stretch in our muscles. These sensations create awareness of our body's positioning. The following receptors are most likely responsible for this ability. A) nociceptors B) proprioceptors C) exteroceptors D) interoceptors

B) proprioceptors

6) The two longitudinal ridges on the medulla oblongata where many descending fibers cross over are called the ________. A) lateral horns B) pyramids C) olives D) nuclei

B) pyramids

45) Motion sickness seems to ________. A) result from activation of nausea centers in the brain stem B) result from mismatch between visual and vestibular inputs C) respond best to medication that "boosts" vestibular inputs D) respond best to medication taken after salivation and pallor begins

B) result from mismatch between visual and vestibular inputs

54) During dark adaptation ________. A) the cones are activated B) rhodopsin accumulates in the rods C) the sensitivity of the retina decreases D) the rate of rhodopsin breakdown is accelerated

B) rhodopsin accumulates in the rods

24) Spinal nerves exiting the cord from the level of L4 to S4 form the ________. A) lumbar plexus B) sacral plexus C) thoracic plexus D) femoral plexus

B) sacral plexus

3) Nerve impulses leading to the skeletal muscle carry information to direct movement. The nerve fibers sending these signals will most likely belong to which division of the nervous system? A) sensory (afferent) division B) somatic nervous system C) sympathetic division D) parasympathetic division

B) somatic nervous system

14) If the caudal portion of the neural tube failed to develop properly the ________. A) hindbrain would not be present B) spinal cord may be affected C) cranial nerves would not form D) telencephalon would cease development

B) spinal cord may be affected

40) Transduction refers to conversion of ________. A) receptor energy to stimulus energy B) stimulus energy into energy of a graded potential C) presynaptic nerve impulses to postsynaptic nerve impulses D) afferent impulses to efferent impulses

B) stimulus energy into energy of a graded potential

27) A reflex that causes muscle relaxation and lengthening in response to muscle tension is called a ________. A) flexor reflex B) tendon reflex C) crossed-extensor reflex D) plantar reflex

B) tendon reflex

44) If a motor neuron in the body were stimulated by an electrode placed about midpoint along the length of the axon ________. A) the impulse would move to the axon terminal only, and the muscle contraction would occur B) the impulse would spread bidirectionally C) muscle contraction would occur D) the impulse would move to the axon terminal only

B) the impulse would spread bidirectionally

14) A patient has an injury of the spine and is now suffering from a loss of motor function in his right arm. However, he still has normal sensory function in the arm. Based on this information it is likely that the patient has nervous tissue damage located at ________. A) the dorsal root located at one or more of the cervical vertebra B) the ventral root located at one or more of the cervical vertebra C) the dorsal rootlets located at one of the thoracic vertebra D) spinal nerves of the cervical vertebra

B) the ventral root located at one or more of the cervical vertebra

11) A doctor asks her patient to follow the motion of her finger as she moves it up and down, left and right. Which of the following cranial nerves is not being tested? A) the oculomotor nerve (III) B) the vestibulocochlear nerve (VIII) C) the abducens (VI) D) the trochlear nerve (IV)

B) the vestibulocochlear nerve (VIII)

48) The blind spot of the eye is caused by ________. A) the macula lutea interrupts the nerve pathway B) there are no photoreceptors where the optic nerve leaves the eye C) an absents of cones in the foveae D) more rods than cones within the retina

B) there are no photoreceptors where the optic nerve leaves the eye

28) Which sympathetic fibers form a splanchnic nerve? A) those that synapse with parasympathetic fibers B) those that pass through the trunk ganglion to synapse in collateral or prevertebral ganglia C) those that synapse with somatic fibers D) those that synapse in the same trunk ganglion they entered

B) those that pass through the trunk ganglion to synapse in collateral or prevertebral ganglia

32) The smooth muscle of the digestive viscera is served largely by the ________. A) pelvic nerves B) vagus (X) nerves C) lumbar splanchnic nerves D) cephalic plexus

B) vagus (X) nerves

41) Problems in balance may follow trauma to which nerve? A) trigeminal B) vestibulocochlear C) abducens D) accessory

B) vestibulocochlear

Which areas of this neuron would be classified as receptive regions?

Both A and B

Which of the following statements is true of both membrane potential responses shown in the graphs?

Both responses are examples of graded potentials.

A molecule that carries information across a synaptic cleft is a A) synaptic cleft. B) receiving neuron. C) neurotransmitter. D) synapse. E) sending neuron.

C

Like all cells, the neurons' internal organization dictates its function. Neurons have relatively many mitochondria, an extensive network of rough endoplasmic reticulum and many clusters of ribosomes. These cellular features indicate all of the following EXCEPT ________. A) neurons have a relatively high consumption of oxygen B) neurons must meet a high demand for ATP C) neurons have stable, relatively unchanging internal environments D) neurons produce many proteins

C

In which area of the neuron is an action potential initially generated?

C Graded potentials originating in the dendrites and cell body are integrated (summated) at the axon hillock (C). Membrane potentials above threshold at the hillock will open voltage-gated Na+ channels found in the "trigger zone," producing an action potential that proceeds down the axon.

Which description of synapses is NOT correct? A) Second messengers can activate gene activities in the postsynaptic neuron. B) Ionotropic receptors are ion channels that open and influence the polarization of the membrane. C) Excitatory synapses cause depolarization. D) Direct signaling involves the activation of G Proteins.

C Direct signaling involves opening ion channels that affect the polarization of the membrane

Which of the following membrane regions would have significant numbers of voltage-gated Na and K ion channels?

C and D

32) Which of the following is false or incorrect? A) An excitatory postsynaptic potential occurs if the excitatory effect is greater than the inhibitory effect but less than threshold. B) An inhibitory postsynaptic potential occurs if the inhibitory effect is greater than the excitatory, causing hyperpolarization of the membrane. C) A nerve impulse occurs if the excitatory and inhibitory effects are equal.

C) A nerve impulse occurs if the excitatory and inhibitory effects are equal.

6) Which of the following is the best explanation of how a stimulus' strength is transmitted to the central nervous system from sensory nerves? A) More than one type of receptor will respond to larger stimulus. B) An action potential will increase in strength as stimulus's strength increases. C) Action potential frequency is increased as stimulus' strength increases. D) Action potentials as well as graded potentials are sent to the central nervous system when stimulus strength increases.

C) Action potential frequency is increased as stimulus' strength increases.

15) When a neurotransmitter like acetylcholine is acting in an excitatory manner which of the following is likely a result of the acetylcholine acting on the post synaptic cell? A) Chemically gated chloride channels will open. B) Chemically gated potassium channels will open. C) Chemically gated sodium channels will open. D) Chemically gated sodium channels will be closed.

C) Chemically gated sodium channels will open.

58) Select the correct statement about equilibrium. A) The weight of the endolymph contained within the semicircular canals against the maculae is responsible for static equilibrium. B) Hair cells of both types of equilibrium hyperpolarize only, resulting in an increased rate of impulse transmission. C) Cristae respond to angular acceleration and deceleration. D) Due to dynamic equilibrium, movement can be perceived if rotation of the body continues at a constant rate.

C) Cristae respond to angular acceleration and deceleration.

45) Feeling a gentle caress on your arm would likely involve all of the following except ________. A) Meissner's corpuscles B) hair follicle receptors C) Lamellar corpuscles D) tactile discs

C) Lamellar corpuscles

13) Sympathetic responses generally are widespread because ________. A) preganglionic fibers are short B) inactivation of ACh is fairly slow C) NE and epinephrine are secreted into the blood as part of the sympathetic response D) preganglionic fibers are long

C) NE and epinephrine are secreted into the blood as part of the sympathetic response

18) Dancers will use a technique called "spotting" when they perform spins of the body. By holding their head and eyes on a fixed point in front of them as their body spins they reduce the amount of head spinning and this prevents dizziness. Which of the following is the most likely explanation for why this works? A) This will help to reduce the lateral flection of the head and will prevent hyper polarization or depolarization of the hair cells in the macula. B) This helps keep the motions detected by the eyes congruent (aligned) with the motions sensed by the vestibular apparatus. C) Reducing the inertia of head spin will reduce the flow of endolymph that deflects the hair cells of the crista ampullaris. D) When the eyes send a static vision of stability to the brain, it is tricked into believing the body is still and therefore dizziness will not occur.

C) Reducing the inertia of head spin will reduce the flow of endolymph that deflects the hair cells of the crista ampullaris.

57) Which of the following is true about photoreceptors? A) In dim light, images are focused directly on the rods in the fovea centralis. B) Three types of color-sensitive photoreceptors exist: red, green, and yellow. C) Rods absorb light throughout the visual spectrum but confer only gray tone vision. D) If all cones are stimulated equally, all colors are absorbed by the cones and the color perceived is black.

C) Rods absorb light throughout the visual spectrum but confer only gray tone vision.

35) Select the correct statement about olfaction. A) Olfactory receptors have a high degree of specificity toward a single type of chemical. B) Substances must be volatile and hydrophobic in order to activate olfactory receptors. C) Some of the sensation of olfaction is actually one of pain. D) Olfactory adaptation is only due to fading of receptor cell response.

C) Some of the sensation of olfaction is actually one of pain.

4) Which of the following is the best explanation for why night vision is fuzzy and indistinct? A) Rods contain a single kind of visual pigment. B) Cones come in three types, each sensitive to different wavelengths of light. C) The foveae are densely packed with cones. D) As many as 100 cones may converge on one ganglion cell.

C) The foveae are densely packed with cones.

33) Parasympathetic functions include ________. A) allowing the body to cope with an external threat B) raising blood glucose levels C) lens accommodation for close vision D) a stimulation of heart rate and force of contraction

C) lens accommodation for close vision

52) Which of the following could not be seen as one looks into the eye with an ophthalmoscope? A) fovea centralis B) macula lutea C) optic chiasma D) optic disc

C) optic chiasma

13) Humans can smell as many as 10,000 different odors but have significantly fewer types of olfactory receptors. Which of the following is the best explanation for why humans can distinguish so many smells? A) The belief that we can smell so many different distinct odors is a psychological process referred to as an uncinate fit or olfactory hallucination. B) The olfactory pathway travels to location in the brain in which memories are formed and we simply mix this new sensory information with old memories. C) The sensation of a single, distinct smell is a combination of a variety of chemicals that stimulate different combinations of olfactory receptor cells all at once. D) Taste receptors that are active at the same time influence the subtlety of what we smell.

C) The sensation of a single, distinct smell is a combination of a variety of chemicals that stimulate different combinations of olfactory receptor cells all at once.

39) Which statement about malnutrition-induced night blindness is most accurate? A) The impaired vision is caused by reduced cone function. B) Visual pigment content is reduced in cones more than rods. C) Vitamin supplements can reverse degenerative changes. D) The most common cause is vitamin D deficiency.

C) Vitamin supplements can reverse degenerative changes.

14) If a post synaptic cell is stimulated to threshold by temporal summation this implies that ________. A) the postsynaptic cell can be influenced by only one presynaptic cell B) the postsynaptic cell is sending frequent action potential C) a presynaptic neuron is sending frequent EPSP D) the presynaptic neuron is sending frequent IPSP

C) a presynaptic neuron is sending frequent EPSP

2) In a person who is color blind, which of the following would you most expect to see? A) a loss of functions in the rods of their retina B) an inability to regenerate 11-cis-retinal after bleaching C) absence of green or red cones in their foveae D) a loss of their peripheral vision

C) absence of green or red cones in their foveae

2) The arbor vitae refers to ________. A) cerebellar gray matter B) flocculonodular nodes C) cerebellar white matter D) the pleatlike convolutions of the cerebellum

C) cerebellar white matter

64) Which of the following is not a possible cause of conduction deafness? A) impacted cerumen B) otosclerosis C) cochlear nerve degeneration D) middle ear infection

C) cochlear nerve degeneration

49) The large commissure that connects the right and left sides of the brain is called the ________. A) corona radiata B) internal capsule C) corpus callosum D) longitudinal fissure

C) corpus callosum

27) Nerve fibers from the medial aspect of each eye ________. A) pass posteriorly without crossing over at the chiasma B) divide at the chiasma, with some crossing and some not crossing C) cross over to the opposite side at the chiasma D) go to the superior colliculus only

C) cross over to the opposite side at the chiasma

32) In a crossed-extensor reflex, if the right arm was grabbed it would flex and the left arm would ________. A) abduct B) also flex C) extend D) adduct

C) extend

14) Sympathetic nerves may leave the spinal cord at which vertebra? A) first coccyx B) third lumbar C) first thoracic D) second cervical

C) first thoracic

21) Bipolar neurons are commonly ________. A) motor neurons B) called neuroglial cells C) found in the retina of the eye D) found in ganglia

C) found in the retina of the eye

16) The sensation of loudness or the volume of a sound is detected by ________. A) high volume sounds can travel all the way the apex of the scala vestibule with enough energy remaining to deflect hair cells B) faster vibration of the basilar membrane resulting in a higher frequency of hair cell stimulation C) greater movement of the basilar membrane resulting in greater deflection of the hair cells D) vibration along a greater length of the basilar membrane, stimulating a greater number of hair cells

C) greater movement of the basilar membrane resulting in greater deflection of the hair cells

60) The receptor membranes of gustatory cells are ________. A) taste buds B) basal cells C) gustatory hairs D) fungiform papillae

C) gustatory hairs

37) An inhibitory postsynaptic potential (IPSP) is associated with ________. A) lowering the threshold for an action potential to occur B) a change in sodium ion permeability C) hyperpolarization D) opening of voltage-regulated channels

C) hyperpolarization

9) Control of temperature, endocrine activity, and thirst are functions associated with the ________. A) thalamus B) medulla C) hypothalamus D) cerebellum

C) hypothalamus

19) Which of the following is NOT a role of the basal nuclei? A) playing a role in cognition and emotion B) controlling starting and stopping movements C) initiating protective reflex actions D) inhibiting unnecessary or antagonistic movements

C) initiating protective reflex actions

33) Which of the following is NOT a function of the CSF? A) nourishment of the brain B) reduction of brain weight C) initiation of some nerve impulses D) protection from blows

C) initiation of some nerve impulses

Chemical synapses are characterized by all of the following except: A) the release of neurotransmitter by presynaptic membranes B) postsynaptic membranes bearing receptors that bind neurotransmitter C) ions flowing through protein channels from the presynaptic to the post synaptic neuron D) a fluid-filled gap separating the neurons

C) ions flowing through protein channels from the presynaptic to the postsynaptic neuron

24) Broca's area ________. A) controls voluntary movements of the eyes B) serves the recognition of complex objects C) is considered a motor speech area D) is usually found only in the right hemisphere

C) is considered a motor speech area

24) Erection (vasodilation) of the penis or clitoris ________. A) is the result of coordinated activation by both sympathetic and parasympathetic input B) is primarily under sympathetic control C) is primarily under parasympathetic control D) depends very little on autonomic activation

C) is primarily under parasympathetic control

22) The frontal lobe is separated from the temporal lobe by the ________. A) longitudinal fissure B) cranial fossa C) lateral sulcus D) central sulcus

C) lateral sulcus

41) Which of the following is mismatched? A) sympathetic preganglionic fibers; release ACh B) parasympathetic preganglionic fibers; release ACh C) parasympathetic postganglionic fibers; release NE D) sympathetic postganglionic fibers; release NE

C) parasympathetic postganglionic fibers; release NE

26) Which of the following structures is not part of the external ear? A) external acoustic meatus B) tympanic membrane C) pharyngotympanic tube D) pinna

C) pharyngotympanic tube

51) Olfactory cells and taste buds are normally stimulated by ________. A) stretching of the receptor cells B) movement of a cupula C) substances in solution D) the movement of otoliths

C) substances in solution

50) Degeneration of the dopamine-releasing neurons of the ________ is the ultimate cause of Parkinson's disease. A) reticular formation B) internal capsule C) substantia nigra D) red nucleus

C) substantia nigra

5) A person picks up a heavy suitcase in order to estimate its weight and reflexively drops it. Which of the following receptors has initiated this reflex? A) lamellae corpuscle B) free nerve ending C) tendon organ D) bulbous corpuscle

C) tendon organ

45) Second-order neurons of ascending pathways that contribute to sensory perception terminate in the ________. A) somatosensory cortex B) spinal cord C) thalamus D) medulla

C) thalamus

9) Cell bodies of sensory neurons are located in ________. A) the thalamus B) sympathetic ganglia C) the dorsal root ganglia of the spinal cord D) the ventral root ganglia of the spinal cord

C) the dorsal root ganglia of the spinal cord

12) As a cook chops red onions he begins to tear up due to activation of the lacrimal gland. Which of the following nerves provided the stimulus? A) the vagus nerve (X) B) the optic nerve (II) C) the facial nerve (VII) D) the olfactory nerve (I)

C) the facial nerve (VII)

15) Injury to cervical vertebra C3-C4 is particularly problematic because ________. A) several ganglia are near this region that serve the heart B) the greater auricular nerve that serve the parotid gland receive there fibers from here C) the phrenic nerve that serves the diaphragm receives its fibers from here D) part of the brain stem is located here

C) the phrenic nerve that serves the diaphragm receives its fibers from here

35) Important nuclei of the indirect (multineuronal) system that receive impulses from the equilibrium apparatus of the inner ear and help to maintain balance by varying muscle tone of postural muscles are the ________. A) red nuclei B) reticular nuclei C) vestibular nuclei D) superior colliculi

C) vestibular nuclei

47) Most newborns ________. A) are myopic B) see in tones of red and green only C) cry with copious tears D) often use only one eye at a time

D) often use only one eye at a time

Motor (efferent) Neuron

Conducts impulses from the brain or spinal cord out to muscles and glands

What type of conduction takes place in unmyelinated axons?

Continuous conduction

How do neurofibrils differ from nerve fibers? A) Neurofibrils are confined to dendrite receptor areas and are consequently not found in nerve fibers. B) There is no real difference, since they are both capable of impulse conduction away from a neuron. C) Neurofibrils are axon subunits that are bundled together to make up a single nerve fiber. D) Neurofibrils are cytoskeletal intermediate filaments maintaining cell shape, but they do not conduct impulses.

D

Prozac belongs to a class of drugs known as selective serotonin reuptake inhibitors (SSRIs), which block the removal of serotonin from its receptor, thereby enhancing a patient's mood. What mechanism allows serotonin to remain on its receptors longer? A) SSRIs are not as easily degraded by acetylcholinesterase, which allows them to remain on their receptors for an extended period of time. B) SSRIs cause an increased number of serotonin receptors expressed on the presynaptic neuron membrane. C) SSRIs prevent the release of serotonin by making a permanent neurotransmitter-receptor complex on the postsynaptic membrane. D) SSRIs inhibit the reuptake of serotonin back into the presynaptic neuron, which increases the level of serotonin in the synaptic cleft.

D

When neurotransmitter molecules bind to receptors in the plasma membrane of the receiving neuron A) vesicles in the synaptic terminal fuse to the plasma membrane of the sending neuron. B) ion channels in the plasma membrane of the sending neuron open. C) the receiving neuron becomes more positive inside. D) ion channels in the plasma membrane of the receiving neuron open. E) the receiving neuron becomes more negative inside.

D

Which of the cell types shown is most associated with the production and flow of cerebrospinal fluid (CSF

D These cells line central cavities of the CNS and, in certain places, produce CSF. The cilia of these cells help circulate the CSF that nourishes and cushions the brain and spinal cord.

10) An action potential is regarded as an example of a positive feedback. Which of the following examples below best illustrates the positive feedback aspect of an action potential? A) The sodium potassium pump consistently moves ions as long as ATP is available, and regardless of membrane potential changes. B) Potassium permeability is about 25 times greater than sodium ions. C) Voltage gated potassium ion channels open slowly and remain open long enough to cause hyperpolarization. D) A threshold stimulus will cause the opening of voltage gated sodium ion channels that will cause further depolarizing stimulus. This stimulus will open still more voltage gated sodium ion channels.

D) A threshold stimulus will cause the opening of voltage gated sodium ion channels that will cause further depolarizing stimulus. This stimulus will open still more voltage gated sodium ion channels.

23) Which of the following statements is FALSE? A) Damage to the visual association area can result in blindness. B) Damage to the premotor cortex results in loss of motor skills programmed in that area but movement is still possible. C) Damage to the primary auditory cortex results in the inability to interpret pitch, loudness, and location. D) Damage to the primary (somatic) motor cortex results in the loss of both voluntary muscle control and all reflexive contractions.

D) Damage to the primary (somatic) motor cortex results in the loss of both voluntary muscle control and all reflexive contractions.

17) The effect of acetylcholine can be stimulating or inhibiting. Which of the following gives the best explanation for why this is so? A) Acetylcholine has many different forms. B) Acetylcholine has a "dose effect". Larger doses are stimulating while small doses inhibit. C) Postsynaptic cells have a dynamic and changing metabolism and respond differently at different times. D) Different post synaptic cells will have different receptors.

D) Different post synaptic cells will have different receptors.

21) The cranial nerve with a cervical origin (spinal cord) is the ________. A) hypoglossal B) vagus C) glossopharyngeal D) accessory

D) accessory

22) Which of the following is an excitatory neurotransmitter secreted by motor neurons innervating skeletal muscle? A) gamma aminobutyric acid B) cholinesterase C) norepinephrine D) acetylcholine

D) acetylcholine

35) The mushroom poison muscarine can bind to receptors on ________. A) cell bodies and dendrites of parasympathetic postganglionic neurons B) cell bodies and dendrites of sympathetic postganglionic neurons C) the hormone producing cells of the adrenal medulla D) all effector cells stimulated by postganglionic cholinergic fibers

D) all effector cells stimulated by postganglionic cholinergic fibers

4) Where would you NOT find a cholinergeric nicotinic receptor? A) all postganglionic neurons (cell bodies and dendrites) B) adrenal medulla hormone producing cells C) sarcolemma of skeletal muscle cells at neuromuscular junctions D) all parasympathetic target organs

D) all parasympathetic target organs

41) Conscious perception of vision probably reflects activity in the ________. A) superior colliculus B) thalamus C) chiasma D) occipital lobe of the cortex

D) occipital lobe of the cortex

8) Overlap in the visual fields of our eyes ________. A) leaves a blind spot anterior to the nose and in the lateral fields of vision B) is essentially a waste of brain processing for what is essentially the same image C) gives us higher visual acuity for small detail by doubling the number of photoreceptors that are being stimulated D) allows us to subconsciously estimate the distance of objects based on the different angles the image strikes our two retinas

D) allows us to subconsciously estimate the distance of objects based on the different angles the image strikes our two retinas

7) The subarachnoid space lies between what two layers of meninges? A) dura and epidura B) arachnoid and epidura C) arachnoid and dura D) arachnoid and pia

D) arachnoid and pia

5) Spinocerebellar tracts ________. A) are found in the dorsal columns of the spinal cord B) terminate in the spinal cord C) give rise to conscious experience of perception D) carry information about muscle or tendon stretch to the cerebellum

D) carry information about muscle or tendon stretch to the cerebellum

35) The sciatic nerve is a combination of which two nerves? A) pudendal and posterior femoral cutaneous B) pudendal and common fibular C) posterior femoral cutaneous and tibial D) common fibular and tibial

D) common fibular and tibial

61) Light passes through the following structures in which order? A) vitreous humor, lens, aqueous humor, cornea B) aqueous humor, cornea, lens, vitreous humor C) cornea, vitreous humor, lens, aqueous humor D) cornea, aqueous humor, lens, vitreous humor

D) cornea, aqueous humor, lens, vitreous humor

20) The parasympathetic tone ________. A) causes blood pressure to rise B) prevents unnecessary heart deceleration C) accelerates activity of the digestive tract D) determines normal activity of the urinary tract

D) determines normal activity of the urinary tract

11) Which of the following is NOT a result of parasympathetic stimulation? A) elimination of urine B) salivation C) increased peristalsis of the digestive viscera D) dilation of the pupils

D) dilation of the pupils

46) A patient who received a blow to the side of the skull exhibits the following signs and symptoms on that side of the face: he is unable to close his eye, and the corner of his mouth droops. Which cranial nerve has been damaged? A) hypoglossal B) accessory C) glossopharyngeal D) facial

D) facial

61) Which of the following is a dural septum, described as a large sickle-shaped fold that dips into the longitudinal fissure between the cerebral hemispheres? A) tentorium cerebelli B) corpus callosum C) falx cerebelli D) falx cerebri

D) falx cerebri

48) Which ventricle is continuous with the central canal of the spinal cord? A) first B) second C) third D) fourth

D) fourth

20) What does the central nervous system use to determine the strength of a stimulus? A) size of action potentials B) origin of the stimulus C) type of stimulus receptor D) frequency of action potentials

D) frequency of action potentials

39) When a sensory neuron is excited by some form of energy, the resulting graded potential is called a(n) ________. A) action potential B) excitatory potential C) postsynaptic potential D) generator potential

D) generator potential

21) Ridges of tissue on the surface of the cerebral hemispheres are called ________. A) fissures B) ganglia C) sulci D) gyri

D) gyri

59) The ________ is the main switch station for memory; if the right and left areas are destroyed, the result is widespread amnesia. A) hypothalamus B) Wernicke's area C) thalamus D) hippocampus

D) hippocampus

31) The first "way station" in the visual pathway from the eye, after there has been partial crossover of the fibers in the optic chiasma, is the ________. A) visual cortex B) superior colliculi C) temporal lobe D) lateral geniculate body of the thalamus

D) lateral geniculate body of the thalamus

10) Which fissure separates the cerebral hemispheres? A) lateral fissure B) central fissure C) parieto-occipital fissure D) longitudinal fissure

D) longitudinal fissure

15) Labyrinthitis is a medical condition often caused by viral infection, resulting in swelling and inflammation of the membranous labyrinth. Which of the following symptoms would you most associate with the condition of labyrinthitis? A) auditory hallucination B) loss of hearing C) ringing in the ears (tinnitus) D) loss of balance and dizziness

D) loss of balance and dizziness

2) Some large arteries that are proximal to the heart are sensitive to the stretch of the blood vessels. This stretch indicates the blood's pressure. Which of the following pairs of classifications below best fit the receptor type that is being described above? A) thermoreceptors that are also exteroceptors B) thermoreceptors that are also interoceptors C) mechanoreceptors that are also exteroceptors D) mechanoreceptors that are also interoceptors

D) mechanoreceptors that are also interoceptors

26) The blood-brain barrier is effective against ________. A) alcohol B) anesthetics C) nutrients such as glucose D) metabolic waste such as urea

D) metabolic waste such as urea

39) Which type of drug would be useful in dilating the pupils for an examination of the retina? A) alpha-1 receptor inhibitor B) beta-3 receptor mimic C) beta-1 receptor mimic D) muscarinic receptor inhibitor

D) muscarinic receptor inhibitor

43) The interior surface of a neuron's plasma membrane at resting membrane potential will have a ________. A) negative charge and contains more sodium than outside of the cell B) positive charge and contains less sodium than outside of the cell C) positive charge and contains more sodium than outside of the cell D) negative charge and contains less sodium than outside of the cell

D) negative charge and contains less sodium than outside of the cell

9) Like all cells, the neurons' internal organization dictates its function. Neurons have relatively many mitochondria, an extensive network of rough endoplasmic reticulum and many clusters of ribosomes. These cellular features indicate all of the following EXCEPT ________. A) neurons produce many proteins B) neurons must meet a high demand for ATP C) neurons have a relatively high consumption of oxygen D) neurons have stable, relatively unchanging internal environments

D) neurons have stable, relatively unchanging internal environments

7) A patient is suffering from the inability to distinguish various types of odors. This patient may have damage to which of the following? A) vagus nerve (X) B) hypoglossal nerve (XII) C) facial nerve (VIII) D) olfactory nerve (I)

D) olfactory nerve (I)

35) Schwann cells are functionally similar to ________. A) astrocytes B) microglia C) ependymal cells D) oligodendrocytes

D) oligodendrocytes

32) As sound levels increase in the spiral organ (of Corti), ________. A) inner hair cells stiffen the basilar membrane B) outer hair cells bend the cilia away from the kinocilium C) inner hair cells bend the cilia away from the kinocilium D) outer hair cells stiffen the basilar membrane

D) outer hair cells stiffen the basilar membrane

1) Nerve impulses are sent to slow the heart's rate of contraction. The nerve fibers sending these signals will most likely belong to which division of the nervous system? A) sensory (afferent) division B) somatic nervous system C) sympathetic division D) parasympathetic division

D) parasympathetic division

2) Preparing the body for the "fight-or-flight" response is the role of the ________. A) somatic nervous system B) parasympathetic division C) cerebrum D) sympathetic division

D) sympathetic division

34) All processing at the circuit level going up to the perceptual level must synapse in the ________. A) medulla B) reticular formation C) pons D) thalamus

D) thalamus

30) Saltatory conduction is made possible by ________. A) erratic transmission of nerve impulses B) diphasic impulses C) large nerve fibers D) the myelin sheath

D) the myelin sheath

3) If a post synaptic neuron is stimulated to threshold by spatial summation this implies that ________. A) the post synaptic cell has a lower than normal threshold B) the post synaptic cell has many voltage gated ion channels C) the post synaptic cell is slow to repolarize D) the postsynaptic cells has many synapses with many presynaptic neurons

D) the postsynaptic cells has many synapses with many presynaptic neurons

8) A patient has lost vision on the left side of both eyes. The patient has likely suffered damage to ________. A) the optic chiasm B) the retinas of the eyes C) the optic nerves D) the right optic tract

D) the right optic tract

19) Visceral reflex arcs differ from somatic in that ________. A) visceral arcs contain two sensory neurons B) somatic arcs contain one additional component that visceral arcs do not possess C) visceral arcs do not use integration centers D) visceral arcs involve two motor neurons

D) visceral arcs involve two motor neurons

46) The only special sense not fully functional at birth is the sense of ________. A) equilibrium B) smell C) taste D) vision E) hearing

D) vision

30) Where would you NOT find autonomic ganglia? A) unpaired, anterior to spinal cord B) within wall of organ served or close to organ C) paired, beside spinal cord D) within spinal cord

D) within spinal cord

Which is the main receptive portion of the neuron?

Dendrites are the main receptive or input regions, providing an enormous surface area for receiving signals from other neurons.

The autonomic nervous system is under voluntary control; whereas, the somatic nervous system is involuntary.

False: Ans=involtunaray CNS is voluntary

Which criterion is used to functionally classify neurons?

Functional classification groups neurons according to the direction in which the nerve impulse travels relative to the central nervous system. Based on this criterion, there are sensory neurons, motor neurons, and interneurons.

A reflex that causes muscle relaxation and lengthening in response to muscle tension is called a ________.

Golgi-tendon reflex

In which area of the neuron is an action potential initially generated?

Graded potentials originating in the dendrites and cell body are integrated (summated) at the axon hillock (C). Membrane potentials above threshold at the hillock will open voltage-gated Na+ channels found in the "trigger zone," producing an action potential that proceeds down the axon. ( points to the the end where it connects to the tail )

Schwann cells a. Are only in the brain b. Are only in the spinal cord c. Are throughout the CNS d. Have a myelin sheath and neurilemma

Have a myelin sheath and neurilemma

chemically gated The receptors at C are affected by the binding of a chemical neurotransmitter.

How would the receptors at C best be classified?

C) depolarization leading to action potentials The voltage-gated Na+ channels cause the rapid depolarization phase of the action potential

If a neuron had a mutation that prevented the production of voltage-gated Na+ channels, what function would the neuron NOT be able to accomplish? A) graded depolarization B) graded hyperpolarization C) depolarization leading to action potentials D) resting membrane potentials

E) the receiving neuron is less likely to generate an action potential. If the receiving neuron is more negative inside, it is less likely to generate an action potential.

If a signal from a sending neuron makes the receiving neuron more negative inside, A) the sending neuron becomes more positive inside. B) the receiving neuron is more likely to generate an action potential. C) the receiving neuron immediately generates an action potential. D) the sending neuron becomes more negative inside. E) the receiving neuron is less likely to generate an action potential.

B. The effect of a neurotransmitter is based on the properties of the receptor more than the neurotransmitter.

In a lab you are conducting tests with various chemicals and neurotransmitter receptors. You notice that exposed frog muscle cells depolarize when you add the chemical nicotine to the acetylcholine (ACh) receptors. What does this tell us about the relationship between neurotransmitters and their receptors? ANSWER: A. The response generated by a receptor is very broad in nature. B. The effect of a neurotransmitter is based on the properties of the receptor more than the neurotransmitter. C. Neurotransmitter receptors only bind specifically with their neurotransmitter. D. The effect of a neurotransmitter is based on the properties of the neurotransmitter more than the receptor.

B. parallel processing

In the brain, vision originates in the rods and cones in the retina. Separate regions of the brain decode basic information, like color, shapes, intensity of light, and there are other regions that decode information like position in space, and awareness of patterns. As you use your visual system, all of these regions are working simultaneously. This simultaneous awareness of all regions working at the same time is due to which processing pattern listed below? ANSWER: A. oscillative processing B. parallel processing C. serial processing D. reflexive processing

C Graded potentials originating in the dendrites and cell body are integrated (summated) at the axon hillock (C). Membrane potentials above threshold at the hillock will open voltage-gated Na+ channels found in the "trigger zone," producing an action potential that proceeds down the axon.

In which area of the neuron is an action potential initially generated? A B C D

B) Resting membrane potential (RMP) The resting membrane potential is the baseline potential that can be recorded across the plasma membrane of an excitable cell prior to excitation.

Ions are unequally distributed across the plasma membrane of all cells. This ion distribution creates an electrical potential difference across the membrane. What is the name given to this potential difference? A) Threshold potential B) Resting membrane potential (RMP) C) Positive membrane potential D) Action potential

When acetylcholine is excitatory, what effect does it have on a postsynaptic cell?

It causes sodium channels to open

The membranes of neurons at rest are very permeable to _____ but only slightly permeable to _____.

K+; Na+

A multipolar neuron contains a. One dendrite and many axons b. Many dendrites and one axon c. One dendrite and one axon d. A single process with the dendrite and axon

Many dendrites and one axon

D. The dendrites provide a large surface area for connections from other neurons.

Many neurons have many short, branching extensions called dendrites. What is the benefit of these structures for a neuron? ANSWER: A. There is a large amount of space for myelin to form and make electrical conduction more efficient. B. There is a large surface area to send signals to other cells. C. There is a large area for production of chemicals used to signal other neurons. D. The dendrites provide a large surface area for connections from other neurons.

E) Ion channel

Match: Ion Channel

Phagocytic neuroglia

Microglia

Which of the following types of glial cells monitors the health of neurons, and can transform into a special type of macrophage to protect endangered neurons?

Microglial cells are small and ovoid with relatively long "thorny" processes. Their processes touch nearby neurons, monitoring their health, and when they sense that certain neurons are injured or are in other trouble, the microglial cells migrate toward them. Where invading microorganisms or dead neurons are present, the microglial cells transform into a special type of macrophage that phagocytizes the microorganisms or neuronal debris.

What is the function of an astrocyte? (5)

Most abundant type- - Provide structural support - Communicate with other neurons - Remove excess ions and neurotransmitters - Transport of substances between blood vessels and neurons -Form scar tissue to replace damaged neurons

Multipolar, Bipolar, or Unipolar? Interneuron

Multipolar

(1.) Contrast unipolar, bipolar and multipolar neurons structurally. (2.) Indicate where each is most likely to be found.

Multipolar neurons have many dendrites, one axon, and are found in the CNS, and motor divisions of the PNS. Bipolar neurons have one axon and one dendrite, and are found in receptor end organs of the special senses such as the retina of the eye and olfactory mucosa. Unipolar neurons are associated with the sensory division of the PNS, and have one process that begins at dendrites that converge directly at the axon. The cell body is found in a dorsal root ganglion or cranial nerve ganglion near the CNS.

In which type of axon will velocity of action potential conduction be the fastest?

Myelinated axons with the largest diameter

The cell body of a neuron contains the a. Nucleus b. Dendrites c. Axon d. Neuroglia

Nucleus

Myelin-forming neuroglia in brain and spinal cord

Oligodendrocyte

Neuroglia that produce myelin insulation in the CNS are a. Microglia b. Astrocytes c. Ependymal cells d. Oligodendrocytes

Oligodendrocytes

Multipolar neuron

One axon and multiple dendrites

A. True

Opening K+ or Cl- channels in a postsynaptic membrane would produce an inhibitory postsynaptic potential (IPSP). ANSWER: A. True B. False

What is the axon hillock?

Origin region of axon

How does the myelination process differ in the CNS and the PNS?

PNS: myelin sheaths in PNS are formed by Schwann cells. Both myelinated and unmyelinated are found in the PNS. Schwann cells form only one segment (internode) of a myelin sheath. The smallest-diameter axons are unmyelinated. These unmyelinated axons are covered by the long extensions of adjacent glial cells. CNS: myelin sheaths in CNS are formed by oligodendrocytes. Both myelinated and unmyelinated are found in the CNS. Oligodendrocytes have multiple flat processes that can coil around as many 60 axons at the same time. Nodes of Ranvier are present, but are more widely spaced than in the PNS. CNS myelin sheaths lack a neurilemma because cell extensions are doing the coiling and the squeezed-out cytoplasm is forced not peripherally but back toward the centrally located nucleus.

May be involved in exacting

Parallel after-discharge

may be involved in exacting types of mental activity

Parallel after-discharge

__________ is a progressive degenerative disease of the basal nuclei that affects the dopamine-secreting pathways.

Parkinson's disease

Impulses continue around and around the circuit until one neuron stops firing

Reverberating

What types of cells are found in the PNS (Peripheral nerves)?

Schwann cells & satellite cells

Since all AP's generated by a given nerve fiber have the same magnitude, how does the CNS "know" whether a stimulus is strong or weak?

The CNS determines a stimulus to be strong when the frequency, or rate, of action potential generation is high. Conversely, a low frequency, or rate, of action potential generation indicates weaker stimuli.

neuroglial cell types are found in the peripheral nervous system (PNS)?

The cell shown in E wraps and insulates the soma of neurons within ganglia in the PNS.

Which areas of this neuron would be classified as receptive regions?

The dendrites and soma of the cell receive signals from other neurons. E would be considered a secretory region, delivering signals to other neurons.

Many neurons have many short, branching extensions called dendrites. What is the benefit of these structures for a neuron?

The dendrites provide a large surface area for connections from other neurons. Yes, because of the branching and extensive membrane surface area, there is a large amount of membrane dedicated to synapses with other neurons.

In a lab you are conducting tests with various chemicals and neurotransmitter receptors. You notice that exposed frog muscle cells depolarize when you add the chemical nicotine to the acetylcholine (ACh) receptors. What does this tell us about the relationship between neurotransmitters and their receptors?

The effect of a neurotransmitter is based on the properties of the receptor more than the neurotransmitter

Which of the following statements most accurately describes the effects caused by binding of the ligand shown to the structure labeled C?

The membrane potential of the postsynaptic membrane changes

What is the function of the myelin sheath?

The myelin sheath increases the speed of action potential conduction from the initial segment to the axon terminals.

What structural classification describes this neuron?

The neuron shown has a many processes (axon and dendrites) that emerge from the cell body. Such neurons typically function as motor neurons or interneurons.

Which of the following occurred in the presence of tetrodotoxin?

The number of action potentials decreased.

C) There are many more K+ leak channels than Na+ leak channels in the plasma membrane. More leak channels translates into more leakiness. Thus the outward flux of K+ is greater than the inward flux of Na+.

The plasma membrane is much more permeable to K+ than to Na+. Why? A) Ligand-gated cation channels favor a greater influx of Na+ than K+. B) There are many more voltage-gated K+ channels than voltage-gated Na+ channels. C) There are many more K+ leak channels than Na+ leak channels in the plasma membrane. D) The Na+-K+ pumps transport more K+ into cells than Na+ out of cells.

Which of the following statements about receptor potentials is FALSE?

The receptor potential is carried by neuroglia.

E) synaptic cleft.

The small space between the sending neuron and the receiving neuron is the A) vesicle. B) calcium channel. C) synaptic terminal. D) neurotransmitter. E) synaptic cleft.

C) Repolarization

The specific period during which potassium ions diffuse out of the neuron d/t a change in membrane permeability A) Action potential B) Relative refractory period C) Repolarization D) Absolute refractory period E) Depolarization

Select the correct statement regarding chemical synapses.

The synaptic cleft uses the action potential to transmit a chemical signal to the post synaptic cell.

Local anesthetics block voltage-gated Na+ channels, but they do not block mechanically gated ion channels. Sensory receptors for touch (and pressure) respond to physical deformation of the receptors, resulting in the opening of specific mechanically gated ion channels. Why does injection of a local anesthetic into a finger still cause a loss of the sensation of touch from the finger?

Touch stimulation of this sensory receptor will open the mechanically gated ion channels, but action potentials are still not initiated because propagation of an action potential requires the opening of voltage-gated Na+ channels.

D) The membrane potential changes from a negative value to a positive value. The plasma membrane, which was polarized to a negative value at the RMP, depolarizes to a positive value

What characterizes depolarization, the first phase of the action potential? A) The membrane potential reaches a threshold value and returns to the resting state. B) The membrane potential changes to a less negative (but not a positive) value. C) The membrane potential changes to a much more negative value. D) The membrane potential changes from a negative value to a positive value.

D) Once the membrane depolarizes to a peak value of +30 mV, it repolarizes to its negative resting value of -70 mV. The plasma membrane was depolarized to a positive value at the peak of the first phase of the action potential. Thus, it must repolarize back to a negative value.

What characterizes repolarization, the second phase of the action potential? A) As the membrane repolarizes to a negative value, it goes beyond the resting state to a value of -80 mV. B) Before the membrane has a chance to reach a positive voltage, it repolarizes to its negative resting value of approximately -70 mV. C) Once the membrane depolarizes to a threshold value of approximately -55 mV, it repolarizes to its resting value of -70 mV. D) Once the membrane depolarizes to a peak value of +30 mV, it repolarizes to its negative resting value of -70 mV.

B) frequency of action potentials

What does the central nervous system use to determine the strength of a stimulus? A) origin of the stimulus B) frequency of action potentials C) type of stimulus receptor D) size of action potentials

C) The membrane potential must depolarize from the resting voltage of -70 mV to a threshold value of -55 mV. This is the minimum value required to open enough voltage-gated Na+ channels so that depolarization is irreversible.

What event triggers the generation of an action potential? A) The membrane potential must depolarize from the resting voltage of -70 mV to its peak value of +30 mV. B) The membrane potential must return to its resting value of -70 mV from the hyperpolarized value of -80 mV. C) The membrane potential must depolarize from the resting voltage of -70 mV to a threshold value of -55 mV. D) The membrane potential must hyperpolarize from the resting voltage of -70 mV to the more negative value of -80 mV.

A) Ca2+, which then causes release of neurotransmitter from the axon terminal Calcium ion channels open when the membrane is depolarized, and the inflow of Ca2+ leads to the release of neurotransmitters from synaptic vesicles.

What ion is entering the axon terminal at A, and what effect does it have? A) Ca2+, which then causes release of neurotransmitter from the axon terminal B) neurotransmitter, which then causes the presynaptic neuron to form an action potential C) K+, which then causes increased production of neurotransmitter D) Na+, which then causes repolarization of the axon terminal's membrane

A. There is a separation of positive and negative charges across a membrane.

What is the basic condition when a neuron is described as polarized? ANSWER: A. There is a separation of positive and negative charges across a membrane. B. There are large, protein anions inside the neuron. C. Na+ is found outside of the neuron. D. Na+ is found outside of the neuron and K+ is found inside of the cell.

C) Voltage-gated Na+ channels change shape, and their activation gates open. The activation gates of voltage-gated Na+ channels open very rapidly in response to threshold stimuli. The activation gates of voltage-gated K+ channels are comparatively slow to open.

What is the first change to occur in response to a threshold stimulus? A) Voltage-gated K+ channels change shape, and their activation gates open. B) Voltage-gated Na+ channels change shape, and their inactivation gates close. C) Voltage-gated Na+ channels change shape, and their activation gates open. D) Voltage-gated Ca2+ channels change shape, and their activation gates open.

C) Voltage-gated Na+ channels The activation gates of voltage-gated Na+ channels open, and Na+ diffuses into the cytoplasm.

What opens first in response to a threshold stimulus? A) Ligand-gated Cl- channels B) Voltage-gated K+ channels C) Voltage-gated Na+ channels D) Ligand-gated cation channels

A) a threshold level depolarization The axolemma must be depolarized to threshold in order to generate an action potential.

What type of stimulus is required for an action potential to be generated? A) a threshold level depolarization B) multiple stimuli C) hyperpolarization D) a suprathreshold stimulus

C) they cause vesicles containing neurotransmitter molecules to fuse to the plasma membrane of the sending neuron.

When calcium ions enter the synaptic terminal, A) neurotransmitter molecules are quickly removed from the synaptic cleft. B) the inside of the receiving neuron becomes more positive. C) they cause vesicles containing neurotransmitter molecules to fuse to the plasma membrane of the sending neuron. D) they cause an action potential in the sending neuron. E) the inside of the receiving neuron becomes more negative.

A) ion channels in the plasma membrane of the receiving neuron open.

When neurotransmitter molecules bind to receptors in the plasma membrane of the receiving neuron, A) ion channels in the plasma membrane of the receiving neuron open. B) ion channels in the plasma membrane of the sending neuron open. C) the receiving neuron becomes more negative inside. D) the receiving neuron becomes more positive inside. E) vesicles in the synaptic terminal fuse to the plasma membrane of the sending neuron.

Since all APs generated by a given nerve fiber have the same magnitude, how does the CNS "know" if a stimulus is strong?

When the frequency or rate of action potential generation is low.

During a neurobiology lecture, a professor repeatedly refers to group A and group B fibers, absolute refractory period and myelin sheath gaps. Define these terms.

When the professor discusses group A fibers, he is referring to fibers that have a large diameter, thickmyelin sheaths, and rapid conduction. Group B fibers are lightly myelinated, have intermediate diameters, and have slower conduction velocity. The absolute refractory period refers to the period of time during which the neuron is incapable of responding to another stimulus because the sodium gates are still open or are inactivated. When the professor discusses the myelin sheath gaps, he is referring to an interruption of the myelin sheath between the wrappings of individual Schwann cells or of the oligodendrocyte processes.

A and B Chemically-gated channels are located primarily in the dendrites and cell body which are considered the "receptive" region of the neuron. This is where presynaptic axon terminals will form synapses on the neuron, and therefore are the parts of the neuron that will "receive" the incoming signals. The chemically gated channels are responsible for generating graded potentials, which are also known as synaptic potentials, and can be either EPSPs or IPSPs.

Where would you expect to find chemically-gated channels?

C, D and E Voltage-gated channels, which are triggered to open by depolarization of the membrane, are found ONLY in the axon, which in this image includes region D, but also includes the axon hillock (C) which is the initial segment of the axon, and region E, which is the terminal end of the axon - the axon terminal.

Where would you expect to see voltage-gated channels?

both A and B The dendrites and soma of the cell receive signals form other neurons.

Which areas of this neuron would be classified as receptive regions?

B. the direction in which the nerve impulse travels relative to the central nervous system

Which criterion is used to functionally classify neurons? ANSWER: A. the number of processes extending from the cell body neuron B. the direction in which the nerve impulse travels relative to the central nervous system C. whether the nerve fibers are myelinated or unmyelinated D. whether the neurons are found within the CNS or the PNS

D) astrocytes Astrocytes are the most abundant and diversely functioning glial cells. Examples of their many functions include: anchoring neurons to capillaries, aiding in the exchanges between neurons and blood, guiding the migration of young neurons, and helping control the chemical environment around neurons.

Which glial cells have the most diversity of function? A) oligodendrocytes B) ependymal cells C) Schwann cells D) astrocytes

C) Unipolar

Which neuron is a sensory neuron found in a reflex arc?

C) Unipolar

Which neuron is common only in dorsal root ganglia of the spinal cord and sensory ganglia of cranial nerves?

B) Bipolar (Retina/Sense organs)

Which neuron is never myelinated?

B) Bipolar (Retina/Sense organs)

Which neuron is rare?

B) Bipolar (Retina/Sense organs)

Which neuron would be found in the retina of the eye?

A) Multipolar (CNS)

Which neuron would connect to a muscle?

C) motor fibers that conduct nerve impulses from the CNS to smooth muscle, cardiac muscle, and glands

Which of the choices below describes the ANS? A) sensory neurons that convey information from somatic receptors in the head, body wall, and limbs and from receptors from the special senses of vision, hearing, taste, and smell to the CNS B) sensory and motor neurons that supply the digestive tract C) motor fibers that conduct nerve impulses from the CNS to smooth muscle, cardiac muscle, and glands D) motor fibers that conduct nerve impulses from the CNS to skeletal muscles

B. degree of myelination of the axon

Which of the following is a factor that determines the rate of impulse propagation, or conduction velocity, along an axon? ANSWER: A. length of the axon B. degree of myelination of the axon C. the number of axon collaterals extending from a truncated axon D. whether the axon is located in the central nervous system or in the peripheral nervous system

B. A neuron can have only one axon, but the axon may have occasional branches along its length.

Which of the following is true of axons? ANSWER: A. Axons use chemically gated ion channels to generate graded potentials. B. A neuron can have only one axon, but the axon may have occasional branches along its length. C. Smaller (thinner) axons are more likely to bear myelin sheaths than larger (thicker) axons. D. Neurons can have multiple axons but only one dendrite

C. Schwann cells

Which of the following peripheral nervous system (PNS) neuroglia form the myelin sheaths around larger nerve fibers in the PNS? ANSWER: A. astrocytes B. oligodendrocytes C. Schwann cells D. satellite cells

D. Sodium/Potassium pumps maintain concentration gradients; sodium and potassium move down their concentration gradients through leakage channels.

Which of the following statements is correct regarding the polarization of a neuronal membrane and the formation of a resting membrane potential? ANSWER: A. As Na+ leaks across the membrane, that establishes a negative charge inside the membrane. B. As sodium leaks into the cell, the inside of the cell becomes more negative. C. A resting potential can be generated without the energy as an input. D. Sodium/Potassium pumps maintain concentration gradients; sodium and potassium move down their concentration gradients through leakage channels.

C) The membrane potential of the postsynaptic membrane changes. Binding of the neurotransmitter to the receptor at C causes the opening of receptor-associated ion channels. Depending on the particular types of channels that open, diffusion of certain ions (typically Na+, K+, or Clâˆ') will cause a depolarizing or hyperpolarizing effect on the postsynaptic membrane.

Which of the following statements most accurately describes the effects caused by binding of the neurotransmitter (green dots) to the structure labeled C? A) The membrane potential of the presynaptic membrane changes. B) The neurotransmitter is transported into the postsynaptic neuron. C) The membrane potential of the postsynaptic membrane changes. D) an action potential is generated

E The cell shown in E wraps and insulates the soma of neurons within ganglia in the PNS.

Which of the neuroglial cell types shown are found in the peripheral nervous system (PNS)? A B C E

What prevents the Na+ and K+ gradients from dissipating? Na+-K+ ATPase H+-K+ ATPase Na+ cotransporter Na+ and K+ leaks

Yes! Also known as the Na+-K+ pump, or simply the pump, this transporter moves three Na+ out of the cell and two K+ into the cell for every ATP it hydrolyzes. This pumping action prevents the Na+ and K+ gradients from running down as these ions passively move through leak channels. The Na+ cotransporter, which uses the energy inherent in the Na+ gradient to transport other ions or molecules (for example, Ca2+ and glucose) across the plasma membrane, is an example of a secondary active transporter. Recall that the Na+ gradient is maintained by the action of the Na+-K+ ATPase. Energy that is liberated during ATP hydrolysis is transformed into the energy of the Na+ gradient. Na+ cotransporter mechanisms do not maintain the Na+ and K+ gradients.

The resting membrane potential depends on two factors that influence the magnitude and direction of Na+ and K+ diffusion across the plasma membrane. Identify these two factors

Yes! The concentration gradient and the large number of K+ leak channels allow for rather robust K+ diffusion out of a cell. In contrast, the concentration gradient and the relatively few Na+ leak channels allow for much less Na+ diffusion into a cell.

On average, the resting membrane potential is -70 mV. What does the sign and magnitude of this value tell you?

Yes! The inside surface of the plasma membrane accumulates more negative charge because of the presence of Na+ and K+ gradients and the selective permeability of the membrane to Na+ and K+.

Ions are unequally distributed across the plasma membrane of all cells. This ion distribution creates an electrical potential difference across the membrane. What is the name given to this potential difference? Action potential Resting membrane potential (RMP) Positive membrane potential Threshold potential

Yes! The resting membrane potential is the baseline potential that can be recorded across the plasma membrane of an excitable cell prior to excitation.

A neuron has a resting potential of -70 mV and a threshold voltage of -50 mV. There are currently three active synapses on the neuron's dendrites, each located the same distance from the axon. The potential changes are + 40 mV at synapse 1, + 20 mV at synapse 2, and -10 mV at synapse 3. These synaptic potentials diminish by 50% by the time they reach the trigger zone. Will this neuron produce an action potential at this time? Explain. (Hint: Draw a graph of the voltage changes.) A) No. The -10 mV potential at synapse 3 inhibits the neuron so that it is unable to produce an action potential. B) Yes. + 40 mV + 20 mV - 10 mV = 50 mV, which is the threshold voltage change required by the neuron. C) Yes. At the trigger zone, all potentials have halved and will sum as follows: +20mV (originated at synapse 1), + 10mV (originated at synapse 2), -5mV (originated at synapse 3) sum to produce a total potential change of +25mV. The neuron required a minimum change of +20 mV (-70 mV + 20 mV = -50 mV), so it will produce an action potential. D) No. The sum of all the potential changes must be exactly -50 mV, because that is the threshold voltage for this neuron.

Yes. At the trigger zone, all potentials have halved and will sum as follows: +20mV (originated at synapse 1), + 10mV (originated at synapse 2), -5mV (originated at synapse 3) sum to produce a total potential change of +25mV. The neuron required a minimum change of +20 mV (-70 mV + 20 mV = -50 mV), so it will produce an action potential.

Sodium and potassium ions can diffuse across the plasma membranes of all cells because of the presence of what type of channel? Sodium-potassium ATPases Leak channels Voltage-gated channels Ligand-gated channels

Yes. Leak channels for Na+ and K+ are ubiquitous, and they allow for the diffusion of these ions across plasma membranes. Some ligand-gated channels do permit Na+ and K+ to passively move across plasma membranes, but they are not present in all cells.

D. The neuron would be unable to repolarize.

You discover that a new chemical compound interacts with K+ voltage-dependent channels. What would be the effect on a neuron if the chemical came into contact with the axonal membrane? ANSWER: A. The cell would die. B. The cell would be unable to depolarize. C. The cell would be unable to generate a resting potential. D. The neuron would be unable to repolarize.

An action potential is regarded as an example of a positive feedback. Which of the following examples below best illustrates the positive feedback aspect of an action potential? a) A threshold stimulus will cause the opening of voltage gated sodium ion channels that will cause further depolarizing stimulus. This stimulus will open still more voltage gated sodium ion channels. b) Potassium permeability is about 25 times greater than sodium ions. c) Voltage gated potassium ion channels open slowly and remain open long enough to cause hyperpolarization. d) The sodium potassium pump consistently moves ions as long as ATP is available, and regardless of membrane potential changes.

a

Complicated interlacing of the ventral rami form networks called nerve plexus. The crisscrossing of the nerve fibers from the various spinal nerves is advantageous because ________. a) injury to any single spinal nerve will be less damaging as there is less chance of total loss of innervation to any particular organ b) viruses that infect us by moving through PNS nerves are prevented entry to the CNS c) having several nerve fibers unite enhances sensory function d) having several nerve fibers unite enhances motor function

a

Ependymal cells line many open cavities in the central nervous system (CNS). Ependymal cells have cilia on the side of the cell that face these openings. What is the most likely function of these ciliated cells? a) movement and circulation of cerebrospinal fluid b) exchange of nutrients between the circulatory system and neurons c) act as macrophage cells to destroy microorganisms or neuronal debris d) create myelin sheaths for CNS cells

a

Nerve impulses leading to the brain carry information about cool temperatures on the skin. The nerve fibers sending these signals will most likely belong to which division of the nervous system? a) sensory (afferent) division b) somatic nervous system c) sympathetic division d) parasympathetic division

a

Starting at the spinal cord, the subdivisions of the brachial plexus are (in order): a) roots, trunks, divisions, and cords b) trunks, divisions, cords, and roots c) divisions, roots, trunks, and cords d) roots, divisions, cords, and trunks

a

What is the basic condition when a neuron is described as polarized? a)There is a separation of positive and negative charges across a membrane. b) There are large, protein anions inside the neuron. c) Na+ is found outside of the neuron and K+ is found inside of the cell. d) Na+ is found outside of the neuron.

a

Which criterion is used to functionally classify neurons? a) the direction in which the nerve impulse travels relative to the central nervous system b) the number of processes extending from the cell body neuron c) whether the nerve fibers are myelinated or unmyelinated d) whether the neurons are found within the CNS or the PNS

a

Which of the choices below describes the ANS? a) motor fibers that conduct nerve impulses from the CNS to smooth muscle, cardiac muscle, and glands b) sensory neurons that convey information from somatic receptors in the head, body wall, and limbs and from receptors from the special senses of vision, hearing, taste, and smell to the CNS c) sensory and motor neurons that supply the digestive tract d) motor fibers that conduct nerve impulses from the CNS to skeletal muscles

a

Which of the following describes the nervous system integrative function? a) analyzes sensory information, stores information, makes decisions b) responds to stimuli by gland secretion or muscle contraction c) senses changes in the environment

a

Which of the following is NOT one of the basic functions of the nervous system? a) Release hormones into the bloodstream to communicate with other cells in the body. b) Decode sensory information from the environment. c) Integrate sensory input for decision making. d) Generate direct, electrical signals.

a

Which of the following is a factor that determines the rate of impulse propagation, or conduction velocity, along an axon? a) degree of myelination of the axon b) the number of axon collaterals c) extending from a truncated axon length of the axon d) whether the axon is located in the central nervous system or in the peripheral nervous system

a

The myelin sheath is

a protein covering around axons only formed by Schwann cells a protein covering around axons only

Which of the following describes a change from the resting membrane potential?

a receptor potential, a synaptic potential, or an action potential

Depolarization of a neuron is

a reversal of charges on either side of the cell membrane

Assume that an EPSP is being generated on the dendritic membrane. Which will occur?

a single channel will open, permitting simultaneous flow of NA+ and K+

assume that EPSP is being generated on the dendrite membrane . Which will occur?

a single type of channel will be open permitting simultaneous flow of Na+ and K+

Assume that an EPSP is being generated on the dendritic membrane. Which will occur?

a single type of channel will open, permitting simultaneous flow of Na+ and K+

The all-or-none principle states that

a stimulated neuron will react fully or not at all

Which of the following does NOT describe the ANS?

a system of motor neurons that innervate all muscle cells

Which of the following mechanisms is most significant in returning Na+ and K+ concentrations to resting ionic conditions (from point D to point E)?

active transport by the Na+-K+ pump Maintenance (and restoration) of the resting ion concentrations depends on the Na+-K+ pump. Once gated ion channels are closed, the combined action of the pump and ion leakage (particularly that of K+) establishes a resting membrane potential in a typical neuron of around âˆ'70 mV.

During the relative refractory period

another action potential can be generated provided the stimulus is large enough

Cold sores on the skin of the mouth occur when herpes simplex viruses that are dormant in neural ganglia become active and travel to the skin of the mouth. Which of the following is the mechanism by which these viruses travel from the ganglia (located within the head) to the skin of the mouth?

anterograde transport

Helps regulate the ionic composition of CNS extracellular fluid

astrocyte

helps regulate the ionic composition of CNS extracellular fluid

astrocyte

may help regulate the ionic composition of the extracellular fluid

astrocyte

2) Neuroglia found in the CNS that change the permeability of capillaries in the brain by binding to and surrounding these blood vessels are called ________.

astrocytes

In the axon, the nerve impulses normally travel

away from the cell body

Neurotransmitter substances are stored in vesicles that are located in specialized portions of the

axon

The anatmomical region of a multipolar neuron that as the lowest threshold for generating an AP is the

axon hillock

The anatomical region of a multipolar neuron that has the lowest threshold for generating an AP is the:

axon hillock

The anatomical region of a multipolar neuron that has the lowest threshold for generating an action potential is

axon hillock

Collections of nerve cell bodies in the peripheral nervous system are called ________. a) nerves b) ganglia c) tracts d) nuclei

b

David, an aspiring baseball player, was struck on the left side of his face with a fastball pitch. He was not wearing a safety helmet. His zygomatic arch was crushed, as well as parts of the temporal bone. Following the accident and reconstructive surgery, he noted that his left lower eyelid was still drooping and the corner of his mouth sagged. What nerve damage did he sustain? a) Trigeminal nerve damage on his left side b) Facial nerve damage on his left side c) Oculomotor nerve damage on his left side d) Trigeminal nerve damage on his right side e) Facial nerve damage on his right side

b

If a signal from a sending neuron makes the receiving neuron more negative inside, a)the receiving neuron is more likely to generate an action potential. b) the receiving neuron is less likely to generate an action potential. c) the receiving neuron immediately generates an action potential. d) the sending neuron becomes more positive inside. e) the sending neuron becomes more negative inside.

b

Mrs. Sagalov has recently been diagnosed with Alzheimer's disease. What clinical symptoms is she likely to show? a) She will show difficulty in communicating, forming relationships with others, and responding appropriately to the environment. b) She will show increasing cognitive deficits, including difficulties with memory and attention, and personality changes such as irritability, moodiness, and confusion. c) She will show a persistent tremor, forward-bent walking posture, shuffling gait, and stiff facial expression. d) She will show wild, jerky, and continuously "flapping" movements, and mental deterioration. She will show visual disturbances, problems controlling muscles, speech disturbances, and urinary incontinence.

b

Nerve impulses leading to the skeletal muscle carry information to direct movement. The nerve fibers sending these signals will most likely belong to which division of the nervous system? a) sensory (afferent) division b) somatic nervous system c) sympathetic division d) parasympathetic division

b

On average, the resting membrane potential is -70 mV. What does the sign and magnitude of this value tell you? a) The outside surface of the plasma membrane is much more negatively charged than the inside surface. b) The inside surface of the plasma membrane is much more negatively charged than the outside surface. c) The inside surface of the plasma membrane is much more positively charged than the outside surface. d) There is no electrical potential difference between the inside and the outside surfaces of the plasma membrane.

b

The concentration neurotransmitters in the fluid of the synaptic cleft must be tightly regulated for neurons to function properly. Which of the following cells is most responsible for aiding in this regulation? a) satellite cells b) astrocytes c) Schwann cells d) oligodendrocytes

b

The concentration of ions in the chemical environment surrounding the neurons must be tightly regulated for neurons to function properly. Which of the following cells is most responsible for this? a) satellite cells b) astrocytes c) Schwann cells d) oligodendrocytes

b

The generation of an action potential in a neuron requires the presence what type of membrane channels? Select the best answer. a) leakage channels b) voltage-gated channels c) chemically gated channels d) membrane channels are not required

b

The plasma membrane is much more permeable to K+ than to Na+. Why? a) There are many more voltage-gated K+ channels than voltage-gated Na+ channels. b) There are many more K+ leak channels than Na+ leak channels in the plasma membrane. c) Ligand-gated cation channels favor a greater influx of Na+ than K+. d) The Na+-K+ pumps transport more K+ into cells than Na+ out of cells.

b

What prevents the Na+ and K+ gradients from dissipating? a) H+-K+ ATPase b) Na+-K+ ATPase c) Na+ cotransporter d) Na+ and K+ leaks

b

Which of the following is not characteristic of neurons? a) They have extreme longevity. b) They are mitotic. c) They conduct impulses. d) They have an exceptionally high metabolic rate.

b

Which of the following is not true of graded potentials? a) They are short-lived. b) They increase amplitude as they move away from the stimulus point. c) They can form on receptor endings. d) They can be called postsynaptic potentials.

b

Which of the following is true about the movement of ions across excitable living membranes? a) Ions always move actively across membranes through leakage channels. b) Sodium gates in the membrane can open in response to electrical potential changes. c) Ions always move from an area of higher concentration to an area of lower concentration d) Ions always move passively across membranes.

b

Which of the following statements is correct concerning the spinal cord? a) The white matter contains cell bodies for spinal nuclei. b) Spinal nerves have mixed motor and sensory function. c) Damage to sensory tracts in the spinal cord leads to paralysis. d) Just like the cerebrum, the gray matter is found on the superficial surfaces.

b

Which of the following statements is correct regarding the polarization of a neuronal membrane and the formation of a resting membrane potential? a) As sodium leaks into the cell, the inside of the cell becomes more negative. b) Sodium/Potassium pumps maintain concentration gradients; sodium and potassium move down their concentration gradients through leakage channels. c) A resting potential can be generated without the energy as an input. d) As Na+ leaks across the membrane, that establishes a negative charge inside the membrane.

b

Which of the following membrane regions would have significant numbers of voltage-gated ion channels

both C and D Voltage-gated Na+ and K+ channels allow for the triggering of an action potential at the axon hillock (C) and its propagation down the axon (D).

An emergency medical technician is examining a trauma victim by shining a pen light into her patient's eye. She records the reactivity of the patents pupils as they constrict when stimulated by the light. This test supports which of the following? a) The patient has suffered brain damage. b) The patient has function of the trochlear nerve (IV). c) The patient has function of the oculomotor nerve (III). d) The patient has lost function of the optic nerve (II).

c

Choose the FALSE statement about nerves. a) Nerves can carry only sensory information, only motor information, or a mixture of sensory and motor information. b) Nerve axons are surrounded by a loose connective tissue layer called the endoneurium. c) Axons make up a majority of the matter in a nerve. d) Nerves consist of parallel bundles of myelinated and nonmyelinated axons.

c

During the relative refractory period of an action potential, a larger than normal stimulus is needed to cause another action potential. This is due to the fact that ________. a) the inactivation gates on voltage gated sodium ion channels are closed b) the membrane is now impermeable to all ions c) the voltage gated potassium ion channels remain open long enough to hyperpolarize the axon membrane d) the sodium potassium pump will stop working during relative refractory

c

Saltatory propagation occurs in _________ axons, in which action potentials _________. Select the best answer. a) myelinated; move continuously along the axon toward the axon hillock b) unmyelinated; move from one node of Ranvier to another c) myelinated; move from one node of Ranvier to another d) unmyelinated; spread by depolarizing the adjacent region of the axon membrane

c

Select the statement that is most correct. a) Ganglia are collections of neuron cell bodies in the spinal cord that are associated with efferent fibers. b) The dorsal root ganglion is a motor-only structure. c) Ganglia associated with afferent nerve fibers contain cell bodies of sensory neurons. d) The cell bodies of afferent ganglia are located in the spinal cord.

c

The interior surface of a neuron's plasma membrane at resting membrane potential will have a ________. a) positive charge and contains less sodium than outside of the cell b) positive charge and contains more sodium than outside of the cell c) negative charge and contains less sodium than outside of the cell d) negative charge and contains more sodium than outside of the cell

c

The resting membrane potential depends on two factors that influence the magnitude and direction of Na+ and K+ diffusion across the plasma membrane. Identify these two factors. a) The presence of concentration gradients and voltage-gated channels b) The presence of a resting membrane potential and leak channels c) The presence of concentration gradients and leak channels d) The presence of concentration gradients and Na+-K+ pumps

c

When calcium ions enter the synaptic terminal, a) they cause an action potential in the sending neuron. b) neurotransmitter molecules are quickly removed from the synaptic cleft. c) they cause vesicles containing neurotransmitter molecules to fuse to the plasma membrane of the sending neuron. d) the inside of the receiving neuron becomes more positive. e) the inside of the receiving neuron becomes more negative.

c

Dendrites

carry impulses toward a nerve cell body

axon

conducting region of neuron; generates nerve impulses

We describe the regeneration of the action potential down the membrane of the axon of the neuron as _______.

conduction or propagation

What is the sympathetic division of the autonomic, or involuntary, nervous system.

consists of visceral motor nerve fibers that regulate the activity of smooth muscles, cardiac muscles, and glands. This division is responsible for generating actions required during activity.

Enteric motor neurons are responsible for

contraction of GI tract smooth muscle

many neurons influence a few neurons

converging

Regeneration within the CNS ________. a) typically allows axonal sprouting of 20 mm b) is more successful than with the PNS c) is promoted by growth inhibitors and glial scars d) is prevented due to growth-inhibiting proteins of oligodendrocytes

d

Which of the following does NOT describe conditions that occur during an action potential? a) The Na+/K+ pump reestablishes resting concentration gradients. b)After initial depolarization, K+ now has both an electrical and a chemical gradient drawing it out of the cell. c) Na+ enters the cell through voltage-gated channels, causing a reversal of the resting membrane potential. d) Na+ is used to repolarize the membrane.

d

Which of the following is NOT a correctly matched pair? a) spinal cord: inner gray matter b) superficial in the brain: gray matter c) gray matter: location of brain nuclei d) gray matter: myelinated axons

d

Which of the following is NOT a function of the hypothalamus? a) regulate food intake b) regulate body temperature c) regulate emotional responses d) regulate the thalamus

d

Which of the following is NOT correct concerning nerves? a) Nerves are analogous to tracts in the CNS. b) Nerves are covered by an outer sheath called the epineurium. c) Nerves do not contain cell bodies. d) Nerves are collection of axons of either sensory or motor neurons but not both

d

Which of the following is not a function of astrocytes? a) control the chemical environment around neurons b) support and brace neurons c) anchor neurons to blood vessels d) provide the defense for the CNS e) guide the migration of young neurons, synapse formation, and helping to determine capillary permeability

d

Which of the following is not a function of the autonomic nervous system? a) innervation of cardiac muscle b) innervation of glands c) innervation of smooth muscle of the digestive tract d) innervation of skeletal muscle

d

Which of the following is true of axons? a)Smaller (thinner) axons are more likely to bear myelin sheaths than larger (thicker) axons. b) Neurons can have multiple axons but only one dendrite. c) Axons use chemically gated ion channels to generate graded potentials. d) A neuron can have only one axon, but the axon may have occasional branches along its length.

d

Which of the following is true of the cerebral hemispheres of the human brain? a) Nearly the entire surface of the cerebral hemispheres is marked by shallow grooves called gyri. b) Nearly the entire surface of the cerebral hemispheres is marked by elevated ridges called sulci. c) The longitudinal fissure separates the cerebral hemispheres from the cerebellum. d) The cerebral hemispheres account for about 83% of total brain mass.

d

Which of the following types of glial cells monitors the health of neurons, and can transform into a special type of macrophage to protect endangered neurons? a) ependymal cells b) oligodendrocytes c) astrocytes d) microglia

d

Which part of the cerebral cortex is involved in intellect, cognition, recall, and personality? a) combined primary somatosensory cortex and somatosensory association cortex b) limbic association area c) posterior association area d) prefrontal cortex (anterior association area)

d

central nervous system

division of the nervous system that is known as the integration and control center

Interneurons receiving input from sensory neurons are located in the ________.

dorsal horn

When neurotransmitter molecules bind to receptors in the plasma membrane of the receiving neuron, a) ion channels in the plasma membrane of the sending neuron open. b) the receiving neuron becomes more negative inside. c) the receiving neuron becomes more positive inside. d) vesicles in the synaptic terminal fuse to the plasma membrane of the sending neuron. e) ion channels in the plasma membrane of the receiving neuron open.

e

Which connective tissue layer directly surrounds every axon in a nerve?

endoneurium

The neuropeptides that act as natural opiates are:

enkephalins

To reach threshold, the amount of sodium

entering the cell must overcome the potassium exiting

Lines brain cavities

ependymal cell

These cells in the CNS have cilia that move in order to circulate cerebrospinal fluid

ependymal cells

What type of cells line the ventricles of the brain?

ependymal cells

Norepinephrine-releasing fibers are called cholinergic fibers. t or f?

false

A major nerve of the lumbar plexus is the ________.

femoral

Which of these would you NOT find in the cerebral cortex?

fiber tracts

he sciatic nerve is a combination of which two nerves?

fibular and tibial

Damage to the ulnar nerve could result in the inability to ______.

flex the wrist

Which of these terms are correctly matched with their definition or description?

hyperpolarization: membrane potential becomes more negative

Which of the following carries no sensory information?

hypoglossal nerve

Which of the following is responsible for the overall integration of the autonomic nervous system (ANS)?

hypothalamus

Loss of function in the enzyme acetylcholine esterase would result in which of the following?

inability to destroy and remove acetylcholine from the synaptic cleft

Increase in stimulus intensity

increases the frequency of action potential

What component of the reflex arc determines the response to a stimulus?

integration center

The time interval between action potentials is called the

interspike interval

Inborn or intrinsic reflexes are ________.

involuntary

When neurotransmitter molecules bind to receptors in the plasma membrane of the receiving neuron,

ion channels in the plasma membrane of the receiving neuron open.

Which of the following correctly describes a graded potential?

it can have amplitudes of various sizes

An IPSP is inhibitory because

it hyperpolarizes the postsynaptic membrane

An IPSP is inhibitory because:

it hyperpolarizes the postsynaptic membrane

If an increase in extracellular potassium hyperpolarizes a neuron, which of the following would be correct

it would change the membrane potential to a more negative value

The major function of the sodium-potassium exchange pump is to

maintain the concentration gradients of Na + and K+ across the plasma membrane

Spinal nerves are all classified as __________.

mixed nerves

somatic nervous system

motor division of the PNS; conducts impulses from CNS to skeletal muscle; voluntary nervous system

Which of the choices below describes the ANS?

motor fibers that conduct nerve impulses from the CNS to smooth muscle, cardiac muscle, and glands

motor (efferent) division

motor nerve fibers; conducts impulses from the CNS to effectors (muscles and glands)

Efferent neurons can usually be classified as

motor neurons

Neurons that carry impulses from the CNS to the body are called

motor neurons

What event is depicted in the structure labeled A?

movement of Ca2+ into the interior of the axonal terminus through voltage-gated channels

The interior surface of a neuron's plasma membrane at resting membrane potential will have a

negative charge and contains less sodium than outside of the cell

Temporal Summation

numerous nerve impulses arriving at a synapse at closely timed intervals exert a cumulative effect

Mixed cranial nerves containing both motor and sensory fibers include all except which of the following?

olfactory

patient is suffering from the inability to distinguish various types of odors. This patient may have damage to which of the following?

olfactory nerve

myelinates nerve fibers in the CNS

oligodendrocyte

mylelinates never fibers in the CNS

oligodendrocyte

In multiple sclerosis, the cells that are the target of an autoimmune attack are the _________.

oligodendrocytes

In multiple sclerosis, the cells that are the target of an autoimmune attack are the

oligodendrocytes Oligodendrocytes are a type of neuroglial cell that function to form the myelin sheath around the axons of neurons within the central nervous system

Nerve impulses are sent to slow the heart's rate of contraction. The nerve fibers sending these signals will most likely belong to which division of the nervous system?

parasympathetic division

Which of the following nerves does not arise from the brachial plexus?

phrenic

Loss of ability to perform skilled motor activities such as piano playing, with no paralysis or weakness in specific muscles, might suggest damage to the ________.

premotor cortex

integration

processing and interpretation of sensory input

A subthreshold stimulus

produces a local potential

For our motor commands to travel toward our muscles, the signals must travel on __________.

projection fibers

Which type of white matter fiber tract connects the cerebrum to lower centers, like the spinal cord?

projection fibers

myelin

protects and electrically insulates axon; increases speed of nerve impulse transmission

Which of the following is NOT an autonomic nervous system (ANS) function?

reflex of skeletal muscles

Which of the following most accurately describes the involvement of the structure labeled B in synaptic signaling?

release of neurotransmitter into the synaptic cleft by exocytosis

Ions are unequally distributed across the plasma membrane of all cells. This ion distribution creates an electrical potential difference across the membrane. What is the name given to this potential difference?

resting membrane potential

Which neuron circuit pattern is involved in the control of rhythmic activities such as breathing?

reverberating circuit

During depolarization of a neuron, sodium ions

rush into the cell

Gluteal innervation comes from which plexus?

sacral

Spinal nerves exiting the cord from the level of L4 to S4 form the ________.

sacral plexus

The three functions of the nervous system are

sensory input, integration, motor output

neuroglia

small cells that surround and wrap delicate neurons

microglial cells

small, ovoid cells with thorny processes touch and monitory neurons; migrate toward injured neurons; monitor the health of other neurons; can transform to phagocytize microorganisms and neuronal debris; known as the defensive cells in the CNS

1) That part of the nervous system that conducts impulses from the CNS to the skeletal muscles is the ________ nervous system.

somatic

sensory (afferent) division

somatic and visceral sensory nerve fibers; conducts impulses from receptors to the CNS

The part of the nervous system that controls voluntary contractions of skeletal muscles is the

somatic nervous system

Assume that an EPSP is being generated on the dendritic membrane. Which will occur?

specific Na+ channels will open

Local potentials

spread over the plasma membrane in decremental fashion, are not propagated for long distances, are graded, can summate

Which of the following is characteristic of the parasympathetic division?

stimulates secretory activity

Spatial Summation

stimulation of a postsynaptic neuron by many terminals at the same time

A second nerve impulse cannot be generated until

the membrane potential has been reestablished

Saltatory conduction is made possible by

the myelin sheath

Saltatory conduction is made possible by ________.

the myelin sheath

When the stimulus intensity increases, _

the number of action potentials increases

The repolarization phase of an action potential results from __________.

the opening of voltage-gated K+ channels

If a signal from a sending neuron makes the receiving neuron more negative inside,

the receiving neuron is less likely to generate an action potential.

The frequency of action potentials is

the reciprocal of the interspike interval, and measured in Herz

What effect did increasing the extracellular potassium have on the resting membrane potential

the resting membrane potential became less negative

What change in a neuron is being measured in the graph?

the voltage measured across the axon membrane at a specific point as an action potential travels past

Suppose that both stimuli seen in these graphs happened equally at the same time on a postsynaptic membrane as a result of two different synapses. Which of the following best describes the result? A) Temporal summation would occur. B) An action potential would result. C) There would be little or no graded potential. D) No chemically gated ion channels would op

there would be little or o graded potential

When calcium ions enter the synaptic terminal,

they cause vesicles containing neurotransmitter molecules to fuse to the plasma membrane of the sending neuron.

An action potential requires

voltage-gated sodium channels to open and sodium to flow with its electrochemical gradient

central nervous system (CNS), peripheral nervous system (PNS)

what are the divisions of the nervous system

sensory input > integration > motor output

what are the functions of the nervous system

dendrites, axons

what are the two types of neuron processes


Kaugnay na mga set ng pag-aaral

chapter 2 science (Respiratory system)

View Set

Davis Edge Medication Patients (Chp 25)

View Set

7.6I Theoretical and Experimental Probability

View Set

CH 14 The Spinal Cord and Spinal Nerves

View Set

Chapter 14: Intro to Swing Components

View Set